NAVLE Prep Questions and Verified 500 Correct Answers Guaranteed A+ At the First Attempt | Exams Nursing | Docsity (2024)

  • Prepare for your exams

  • Get points

  • Guidelines and tips

  • Sell on Docsity
Log inSign up

NAVLE Prep Questions and Verified 500 Correct Answers Guaranteed A+ At the First Attempt | Exams Nursing | Docsity (2)

NAVLE Prep Questions and Verified 500 Correct Answers Guaranteed A+ At the First Attempt | Exams Nursing | Docsity (3)

Prepare for your exams

Study with the several resources on Docsity

NAVLE Prep Questions and Verified 500 Correct Answers Guaranteed A+ At the First Attempt | Exams Nursing | Docsity (4)

Earn points to download

Earn points by helping other students or get them with a premium plan

NAVLE Prep Questions and Verified 500 Correct Answers Guaranteed A+ At the First Attempt | Exams Nursing | Docsity (5)

Guidelines and tips

Sell on Docsity
Log inSign up

NAVLE Prep Questions and Verified 500 Correct Answers Guaranteed A+ At the First Attempt | Exams Nursing | Docsity (7)

Prepare for your exams

Study with the several resources on Docsity

Find documentsPrepare for your exams with the study notes shared by other students like you on DocsitySearch Store documentsThe best documents sold by students who completed their studies

Search through all study resources

Docsity AINEWSummarize your documents, ask them questions, convert them into quizzes and concept mapsExplore questionsClear up your doubts by reading the answers to questions asked by your fellow students

NAVLE Prep Questions and Verified 500 Correct Answers Guaranteed A+ At the First Attempt | Exams Nursing | Docsity (8)

Earn points to download

Earn points by helping other students or get them with a premium plan

Share documents20 PointsFor each uploaded documentAnswer questions5 PointsFor each given answer (max 1 per day)

All the ways to get free points

Get points immediatelyChoose a premium plan with all the points you need

Study Opportunities

Search for study opportunitiesNEWConnect with the world's best universities and choose your course of study

Community

Ask the communityAsk the community for help and clear up your study doubts University RankingsDiscover the best universities in your country according to Docsity users

Free resources

Our save-the-student-ebooks!Download our free guides on studying techniques, anxiety management strategies, and thesis advice from Docsity tutors

From our blog

Exams and Study

Go to the blog

Walden UniversityNursing

NAVLE Prep Questions and Verified 500 Correct Answers Guaranteed A+ At the First AttemptNAVLE Prep Questions and Verified 500 Correct Answers Guaranteed A+ At the First AttemptNAVLE Prep Questions and Verified 500 Correct Answers Guaranteed A+ At the First Attempt

Typology: Exams

2023/2024

1 / 102

NAVLE Prep Questions and Verified 500 Correct Answers Guaranteed A+ At the First Attempt | Exams Nursing | Docsity (12)

Related documents

(3)

NAVLE Prep Questions and Verified 500 Correct Answers Guaranteed A+ At the First Attempt | Exams Nursing | Docsity (13)

NAVLE Prep Questions and Verified Correct Answers Guaranteed A+2024

NAVLE Prep Questions and Verified 500 Correct Answers Guaranteed A+ At the First Attempt | Exams Nursing | Docsity (14)

NAVLE Prep| Correct Questions| Answers|Verified 100%

NAVLE Prep Questions and Verified 500 Correct Answers Guaranteed A+ At the First Attempt | Exams Nursing | Docsity (15)

NAVLE Prep Questions and Verified 500 Correct Answers Guaranteed A+ At the First Attempt | Exams Nursing | Docsity (16)

NAVLE Prep Questions and Verified 500 Correct Answers Guaranteed A+ At the First Attempt | Exams Nursing | Docsity (17)

AHIP first test Exam questions and answers 100%correct/verified Guaranteed success. [New U

NAVLE Prep Questions and Verified 500 Correct Answers Guaranteed A+ At the First Attempt | Exams Nursing | Docsity (18)

Vet Prep NAVLE Questions with correct Answers 2024

NAVLE Prep Questions and Verified 500 Correct Answers Guaranteed A+ At the First Attempt | Exams Nursing | Docsity (19)

AHIP first test Exam questions and answers 100%correct/verified Guaranteed success New Upd

NAVLE Prep Questions and Verified 500 Correct Answers Guaranteed A+ At the First Attempt | Exams Nursing | Docsity (20)

AHIP first test Exam questions and answers 100%correct/verified Guaranteed success New Upd

NAVLE Prep Questions and Verified 500 Correct Answers Guaranteed A+ At the First Attempt | Exams Nursing | Docsity (21)

AHIP first test Exam questions and answers 100%correct/verified Guaranteed success New Upd

AHIP first test Exam questions and answers 100%correct/verified Guaranteed success New Upd

NAVLE Prep Questions and Verified 500 Correct Answers Guaranteed A+ At the First Attempt | Exams Nursing | Docsity (23)

NUR2868 Exam RS Questions with Verified Correct Answers/A+ Grade Guaranteed 2023

NAVLE Prep Questions and Verified 500 Correct Answers Guaranteed A+ At the First Attempt | Exams Nursing | Docsity (24)

Golani ACT Prep 2022-23 GUARANTEED 100% VERIFIED ANSWERS CORRECT

NAVLE Prep Questions and Verified 500 Correct Answers Guaranteed A+ At the First Attempt | Exams Nursing | Docsity (25)

Exam Questions New Update 2023/2024 (With Attempted Answers And Correct Answers Verified W

NAVLE Prep Questions and Verified 500 Correct Answers Guaranteed A+ At the First Attempt | Exams Nursing | Docsity (26)

ACT Prep Reading Test Questions with Verified Answers,100% CORRECT

NAVLE Prep Questions and Verified 500 Correct Answers Guaranteed A+ At the First Attempt | Exams Nursing | Docsity (27)

ACT Prep Test 44 Questions with Verified Answers,100% CORRECT

NAVLE Prep Questions and Verified 500 Correct Answers Guaranteed A+ At the First Attempt | Exams Nursing | Docsity (28)

NAVLE Prep Questions And Answers.

NAVLE Prep Questions and Verified 500 Correct Answers Guaranteed A+ At the First Attempt | Exams Nursing | Docsity (29)

NAVLE Prep Questions And Answers.

NAVLE Prep Questions and Verified 500 Correct Answers Guaranteed A+ At the First Attempt | Exams Nursing | Docsity (30)

NAVLE Prep Questions And Answers.

NAVLE Prep Questions and Verified 500 Correct Answers Guaranteed A+ At the First Attempt | Exams Nursing | Docsity (31)

NAVLE Questions: Bovine EXAM 2024-2025 UPDATE ACTUAL EXAM ALL QUESTIONS AND CORRECT DETAIL

NAVLE Prep Questions and Verified 500 Correct Answers Guaranteed A+ At the First Attempt | Exams Nursing | Docsity (32)

Custodian Test Prep Class 89 Questions with Verified Answers,100% CORRECT

NAVLE Prep Questions and Verified 500 Correct Answers Guaranteed A+ At the First Attempt | Exams Nursing | Docsity (33)

NAVLE prep237 questions and Answers 2024

NAVLE Prep Questions and Verified 500 Correct Answers Guaranteed A+ At the First Attempt | Exams Nursing | Docsity (34)

NAVLE Prep Questions and Answers 2024

NAVLE Prep Questions and Verified 500 Correct Answers Guaranteed A+ At the First Attempt | Exams Nursing | Docsity (35)

NAVLE Prep Test Questions And Answers.

NAVLE Prep Questions and Verified 500 Correct Answers Guaranteed A+ At the First Attempt | Exams Nursing | Docsity (36)

NAVLE Prep Questions with Answers 2024.

NAVLE Prep Questions and Verified 500 Correct Answers Guaranteed A+ At the First Attempt | Exams Nursing | Docsity (37)

NCLEX RN 2024-2025 VERIFIED QUESTIONSAND CORRECT ANSWERS 100% SUCCESS GUARANTEED

NAVLE Prep Questions and Verified 500 Correct Answers Guaranteed A+ At the First Attempt | Exams Nursing | Docsity (38)

ACT Prep and Final Exam 94 Questions with Verified Answers,100% CORRECT

NAVLE Prep Questions and Verified 500 Correct Answers Guaranteed A+ At the First Attempt | Exams Nursing | Docsity (39)

NAVLE Prep Exam Questions and Answers 2024

NAVLE Prep Questions and Verified 500 Correct Answers Guaranteed A+ At the First Attempt | Exams Nursing | Docsity (40)

AIPMM- CPM EXAM 2023-2024 QUESTIONS AND CORRECT ANSWERS(VERIFIED ANSWERS) 100% Guaranteed

NAVLE Prep Questions and Verified 500 Correct Answers Guaranteed A+ At the First Attempt | Exams Nursing | Docsity (41)

ENPC Test Questions And Answers 100% Verified Correct Rated (A+). Guaranteed Success.

Partial preview of the text

Download NAVLE Prep Questions and Verified 500 Correct Answers Guaranteed A+ At the First Attempt and more Exams Nursing in PDF only on Docsity! NAVLE Prep Questions and Verified 500 Correct Answers Guaranteed A+ At the First Attempt A 1 y/o female stray DSH cat was presented for pruritus. Your physical exam revealed a generalized miliary dermatitis and dark brown flecks in the fur. Which of the following findings are most consistent with the most likely diagnosis? A. Mites found on skin scraping, peripheral eosinophilia, anemia B. Mites found on ear swabbing, peripheral eosinophilia, anemia C. Yeast organisms found on impression smear D. Anemia, peripheral eosinophilia, tapeworm infestation - CORRECT ANSWER: D. Anemia, peripheral eosinophilia, tapeworm infestation A 1-year old female spayed Doberman Pinscher has presented after being hit by a car. Initial chest radiographs show mild contusions, and the patient appears to be otherwise stable. A right mid-shaft long oblique femoral fracture has been identified. Routine pre- operative blood work is unremarkable. A buccal mucosal bleeding test (BMBT) is elevated at 6 minutes. What will you administer prior to surgery? A. 1,25 dihydrocholecalciferol B. Desmopressin acetate C. Vitamin K D. Whole blood transfusion - CORRECT ANSWER: B. Desmopressin acetate A 1-year old female spayed Doberman Pinscher has presented after being hit by a car. Initial chest radiographs show mild contusions, and the patient appears to be otherwise stable. A right mid-shaft long oblique femoral fracture has been identified. Routine pre- operative blood work is unremarkable. A buccal mucosal bleeding test (BMBT) is elevated at 6 minutes. What will you administer prior to surgery? A. Desmopressin acetate B. Vitamin K C. Whole blood transfusion D. 1,25 dihydrocholecalciferol - CORRECT ANSWER: A. Desmopressin acetate A 1-year old mixed breed stray dog is presented for extreme pruritis and the lesion shown in the picture. The dog was recently found and adopted off the street and has been itching despite being bathed and treated with flea preventatives. The owner reports that she has been itching and developing rashes on her own body since adopting this dog. Your physical exam shows that the dog has several additional similar lesions on the other legs, chest, and ventral abdomen. What is the most likely diagnosis? A. Sarcoptes infestation B. Cutaneous lymphoma C. Squamous cell carcinoma D. Demodex infection - CORRECT ANSWER: A. Sarcoptes infestation A 1-year-old female guinea pig is presented with round patches of scaly alopecia on the tip of the nose, ears, periocular areas and forehead. The affected areas do not appear to be pruritic. Skin scrapings from the periphery of lesions and associated broken hairs demonstrate hyphae and arthrospores on direct microscopic examination. Which choice is the most likely diagnosis? A. Cheyletiellosis B. Dermatophytosis C. Demodicosis D. Trombiculosis E. Notoedric mange - CORRECT ANSWER: B. Dermatophytosis A 1-year-old female llama is presented with hyperkeratosis around the nose and mouth. What is the most likely diagnosis? A. Munge B. Dermatophilosis C. Dorsal nasal alopecia D. Wry face E. Zinc-responsive dermatosis - CORRECT ANSWER: A. Munge A 1.5-year old Quarter Horse gelding is presented to you for symmetric ataxia, weakness, and spasticity of all limbs, but worse in the hind limbs. When walking, the horse frequently drags his toes and the hind limbs frequently interfere with one another. Based on the signalment, history and physical examination findings, which of the following is the most likely cause of these clinical signs? A. Botulism B. Cauda Equina Syndrome C. Equine Degenerative Myeloencephalopathy (EDM) D. Equine Motor Neuron Disease (EMND) E. Equine Protozoal Myeloencephalitis (EPM) - CORRECT ANSWER: C. Equine Degenerative Myeloencephalopathy (EDM) A 1.5-year-old spayed female cat presents with a 7-day history of vocalizing, rolling, and allowing a male neutered cat in the household to mount her. The cat has gone through one similar episode 1 month ago. She otherwise has been behaving normally and has no other health problems. You performed an ovariohysterectomy on the cat at 3 months of age. You perform vagin*l cytology which shows some cornified epithelial cells but is inconclusive. You measure serum lutenizing hormone of 0.2 ng/ml (normal for an ovariectomized female is >1 ng/ml). Which of the following is the most appropriate course of action? A. Order MRI of the brain to rule out a pituitary tumor B. Measure serum estrogen and progesterone levels C. Measure serum testosterone levels D. Exploratory laparotomy to remove the ovarian remnant - CORRECT ANSWER: D. Exploratory laparotomy to remove the ovarian remnant A 2 year old Angus cow is presented with weakness of the hind limbs and a staggering gait. On physical exam, the heart rate and respiratory rate are slow and irregular, and the pupils are dilated. The cow's breath and urine smell like the odor of mouse urine. A 2-year old male castrated Border Collie presents for a 1-week history of small bowel diarrhea. A fecal flotation shows numerous Giardia cysts. What is the treatment of choice for this dog? A. Metronidazole B. Decoquinate C. Albendazole D. Ipronidazole - CORRECT ANSWER: A. Metronidazole (Note - if fenbendazole were given as an option, it would be the best choice over metro) A 2-year old male castrated mixed breed dog presents for an altered gait after being hit by a car. On physical exam, the thoracic limbs had decreased biceps and triceps reflexes and decreased muscle tone. The pelvic limbs had hyper-reflexive patellar and gastrocnemius reflexes and increased muscle tone. Where is the spinal cord lesion? A. C6-T2 B. L4-S3 C. T3-L3 D. C1-C5 - CORRECT ANSWER: A. C6-T2 A 2-year old male neutered cat presents to you depressed, hypersalivating, and ataxic with muscle tremors. The owner reports that a pyrethrin-based spot-on formulation for flea control belonging to their Golden Retriever was accidentally applied on the cat earlier today. Which of the following drugs will you use to treat the cat's clinical signs? A. 2-PAM B. Atropine C. Methocarbamol D. Acepromazine E. Amoxicillin - CORRECT ANSWER: C. Methocarbamol A 2-year old male neutered cat presents to you depressed, hypersalivating, and ataxic with muscle tremors. The owner reports that a pyrethrin-based spot-on formulation for flea control belonging to their Golden Retriever was accidentally applied on the cat earlier today. Which of the following drugs will you use to treat the cat's clinical signs? A. Methocarbamol B. Amoxicillin C. Acepromazine D. Atropine E. 2-PAM - CORRECT ANSWER: A. Methocarbamol A 2-year old mare presents to you several weeks after recovering from a mild upper respiratory infection. She now presents with edema and sloughing of the legs (see image), chest and abdomen as well as mucosal petechial hemorrhages. She is sore and reluctant to move. Biopsy of the skin lesion is consistent with aseptic necrotizing vasculitis. What is the most likely diagnosis? A. Cantharidin toxicity B. Bastard strangles C. Type I hypersensitivity D. Purpura hemorrhagica E. Idiopathic thrombocytopenia - CORRECT ANSWER: D. Purpura hemorrhagica A 2-year old mare presents to you several weeks after recovering from a mild upper respiratory infection. She now presents with edema and sloughing of the legs, chest and abdomen as well as mucosal petechial hemorrhages. She is sore and reluctant to move. Biopsy of the skin lesion is consistent with aseptic necrotizing vasculitis. What is the most likely diagnosis? A. Type I hypersensitivity B. Idiopathic thrombocytopenia C. Bastard strangles D. Cantharidin toxicity E. Purpura hemorrhagica - CORRECT ANSWER: E. Purpura hemorrhagica A 2-year old nonlactating dairy cow in California has a sudden onset of head tilt and drooling. The owner also reports that she appears less active and less interested in feed than normal, and today is circling in one direction in the pen. You examine her and find T=105F (40.6 C), HR=96, and RR=32. There is ptosis, drooped ear, and weakness of the lips on the affected side. You take a lumbosacral spinal tap (see photo), and submit the CSF to your lab. The results show elevated protein and WBCs, with the cell type being mainly monocytes. The lab reports seeing some gram positive bacteria in the monocytes of the CSF. Based on these findings what is the best treatment for this condition? A. Metoclopramide B. Chloramphenicol C. Penicillin D. Metronidazole E. Enrofloxacin - CORRECT ANSWER: C. Penicillin Diagnosis is Listeriosis A 2-year old nonlactating dairy cow in California has a sudden onset of head tilt and drooling. The owner also reports that she appears less active and less interested in feed than normal, and today is circling in one direction in the pen. You examine her and find T=105F (40.6 C), HR=96, and RR=32. There is ptosis, drooped ear, and weakness of the lips on the affected side. You take a lumbosacral spinal tap, and submit the CSF to your lab. The results show elevated protein and WBCs, with the cell type being mainly monocytes. The lab reports seeing some gram positive bacteria in the monocytes of the CSF. Based on these findings what is the best treatment for this condition? A. Enrofloxacin B. Penicillin C. Metronidazole D. Chloramphenicol E. Metoclopramide - CORRECT ANSWER: B. Penicillin A 2-year old Quarter Horse presents for intermittent muscle fasciculation followed by weakness. What condition should you suspect? A. Grass tetany B. Stringhalt C. Myotonia D. Hyperkalemic periodic paralysis E. Tetanus - CORRECT ANSWER: D. Hyperkalemic periodic paralysis A 2-year old, male castrated domestic short-hair cat is presented to you for hematuria, strangiuria, and pollakiuria of 3 days duration. The cat has no other historical medical problems. What is the most likely diagnosis? A. Fanconi syndrome B. Feline idiopathic cystitis (FIC) C. Bladder stones D. Transitional cell carcinoma E. Bacterial urinary tract infection - CORRECT ANSWER: B. Feline idiopathic cystitis A 2-year-old cat has a two-day history of anorexia and vomiting. Abdominal radiographs show that the intestines appear bunched up in accordion like pleats. Which of the following is the most likely cause? A. Intestinal adenocarcinoma B. Intestinal lymphosarcoma C. Intestinal parasites D. Intussusception E. A string foreign body - CORRECT ANSWER: E. A string foreign body A 2-year-old cat has a two-day history of anorexia and vomiting. Abdominal radiographs show that the intestines appear bunched up in accordion-like pleats. Which of the following is the most likely cause? A. Intestinal adenocarcinoma B. Intestinal lymphosarcoma C. Intestinal parasites D. Intussusception E. A string foreign body - CORRECT ANSWER: E. A string foreign body A 2-year-old Quarter horse filly presents with a history of intense pruritis and alopecia in the perineal area. The owners indicated that they noticed the filly rubbing her tail head and perineal area along fences for a period of one week. What diagnostic test will you use to confirm your top differential? A. A scotch tape test to confirm Strongyloides vulgaris B. A fecal flotation to confirm Cyathostomiasis C. A superficial skin scrape to confirm Chorioptes spp. D. A scotch tape test to confirm Oxyuris equi E. A fecal flotation to confirm Oxyruis equi - CORRECT ANSWER: D. A scotch tape test to confirm Oxyuris equi A 2-year-old Thoroughbred filly has a 1-month history of poor race performance. The mare slows down markedly near the end of races with labored breathing, excessive bilaterally absent, and the hindlimb toes are cold to the touch. The cat is tachypenic, but lung sounds are normal. What is the most likely diagnosis? A. Lymphoma of the spinal cord B. T3-L3 intervertebral disc disease C. Chlorpyrifos toxicity D. Tetanus E. Thrombus at the aortic bifurcation (saddle thrombus) - CORRECT ANSWER: E. Thrombus at the aortic bifurcation (saddle thrombus) A 3-year old, indoor-only, male castrated Maine Coon cat is presented to you for acute- onset of hindlimb pain and paresis. Physical examination reveals a grade III left parasternal heart murmur and minimal movement in the hind legs. Femoral pulses are bilaterally absent, and the hindlimb toes are cold to the touch. The cat is tachypenic, but lung sounds are normal. What is the most likely diagnosis? A. Tetanus B. Thrombus at the aortic bifurcation (saddle thrombus) C. T3-L3 intervertebral disc disease D. Lymphoma of the spinal cord E. Chlorpyrifos toxicity - CORRECT ANSWER: B. Thrombus at the aortic bifurcation (saddle thrombus) A 3-year-old Quarter horse/Appaloosa cross mare is presented on emergency because she collapsed in the last 30 minutes. The mare is recumbent and weak, with respiratory stridor, muscle fasciculations, sweating, prolapse of the third eyelid and her lips pulled back spasmodically, like a grin. The horse is hydrated and has had no recent history of colic. During the examination, she improves markedly and within 60 minutes stands and appears almost normal again with no treatment. What is the clinical diagnosis? Value / Normal 100.2 F (37.9 C) / 99.0-100.6, 37.2-38.1 C HR=24 bpm / 28-40 bpm BR=16 brpm / 10-14 brpm A. Myasthenia gravis B. Epilepsy C. Ryegrass staggers D. Hyperkalemic periodic paralysis E. Equine degenerative myelopathy - CORRECT ANSWER: D. Hyperkalemic periodic paralysis A 4 -year old domestic short haired cat presents for anorexia and weight loss of 1 week. Physical exam reveals a body condition score of 7/9, jaundice of the skin and sclera, and dehydration of 4%. Temperature is normal. Bloodwork shows: ALT=303 (25- 97 U/L) GGT=1.8 (0-6 U/L) ALP=1170 (0-45 U/L) bilirubin=3.0 (0-0.1 mg/dl) Radiographs show an enlarged liver. What is the most important treatment for the cat's likely diagnosis? A. Oral clavulanic acid and amoxicillin for 4 weeks B. Oral S-adenosylmethionine for at least 1 month C. Esophagostomy tube feeding D. Parenteral vitamin K injections - CORRECT ANSWER: C. Esophagostomy tube feeding -Hepatic lipidosis A 4 month old calf from a small beef herd in Louisiana presents with a fracture of the right metatarsus. He was normal last night.The owner found the calf this morning after rounding up the herd for deworming. This is the 5th calf this season with fractured bones. One had a vertebral fracture, two calves had humeral fractures, and one other had a cannon bone fracture. Which one of the following choices is the most likely problem in this herd? A. Low dietary copper B. High dietary phosphorus C. Hypovitaminosis D D. Low dietary protein E. High dietary calcium - CORRECT ANSWER: A. Low dietary copper A 4 year old mare has bilateral swelling and drainage of the mandibular lymph nodes. Rectal temperature is 101.8F (38.8 C). Which of the following is the most appropriate plan for this horse? A. Administer penicillin B. Administer corticosteroids C. Inform the state veterinarian D. Culture the discharge for bacteria - CORRECT ANSWER: D. Culture the discharge for bacteria A 4 year old mare has bilateral swelling and drainage of the mandibular lymph nodes. Rectal temperature is 101.8F (38.8 C). Which of the following is the most appropriate plan for this horse? A. Inform the state veterinarian B. Administer penicillin C. Administer corticosteroids D. Culture the discharge for bacteria - CORRECT ANSWER: D. Culture the discharge for bacteria A 4-day-old foal is presented with clinical signs of depression, icterus, head pressing, and disorientation. Serum biochemistry reveals increased levels of gamma- glutamyltransferase, alkaline phosphatase, and unconjugated bilirubin. On further questioning, the owner mentions that he gave the foal an oral nutritional supplement when it was a day old but was not sure what was in it. What could this supplement have contained to cause these clinical signs? A. Vitamin E B. Copper C. Iron D. Vitamin A E. Selenium - CORRECT ANSWER: C. Iron A 4-month-old mixed breed puppy is presented with a 1-month history of generalized wobbliness when walking and head bobbing when attempting to eat. Owners do not think there has been progression of the symptoms. Based on the MRI findings, which one of the following choices is the most likely diagnosis? A. Distemper encephalitis B. Lead toxicosis C. Cerebellar hypoplasia D. Chiari malformation E. Normal MRI - CORRECT ANSWER: C. Cerebellar hypoplasia A 4-month-old terrier cross presents for inability to eat. The puppy is bright, alert, and responsive on exam. Heart rate is 148, respiratory rate is panting, and temperature is 101.8 F (38.8 C). The puppy is extremely painful when his lower jaw is palpated, and cries and pulls away when you attempt to open his mouth. Sedated oral exam is unremarkable. Radiographs are available for review. What do you tell the owner about prognosis? A. This will regress within a year, and symptomatic support is needed for discomfort. B. Surgical removal of dentigerous cysts will likely be curative, however long term dental disease is common. C. This can be cured with antibiotics based on culture and sensitivity. NSAIDs can be used for discomfort. D. Chemotherapy can prolong quality of life for a few of months, however prognosis is grave. - CORRECT ANSWER: A. This will regress within a year, and symptomatic support is needed for discomfort. -Craniomandibular osteopathy (young dogs, terriers predisposed) A 4-year old domestic short haired male cat presents for an acute onset of vomiting and anorexia of one day duration. On physical exam you note the cat is depressed and approximately 5% dehydrated. Radiographs show an abnormally increased amount of plication of the small intestines. What is your most likely diagnosis? A. Panleukopenia B. Hepatic lipidosis C. Lymphoma D. Pancreatitis E. String foreign body - CORRECT ANSWER: E. String foreign body A 4-year old female ferret presents with a history of an enlarged vulva and truncal alopecia. What is the most likely diagnosis? A. Adrenal tumor B. Insulinoma C. Sertoli cell tumor D. Hypothyroidism - CORRECT ANSWER: A. Adrenal tumor A 4-year old female ferret presents with a history of an enlarged vulva and truncal alopecia. What is the most likely diagnosis? activated charcoal. The owner returns with a box of rodenticide that says brodifacoum. What should you recommend? A. Treat the dog with vitamin E and selenium B. No additional treatment or monitoring is needed C. Hospitalize the dog for 24 hours to monitor and treat potential neurologic signs D. Treat the dog with vitamin K1 for 6 weeks E. Check serum calcium levels today and once weekly for 6 weeks - CORRECT ANSWER: D. Treat the dog with vitamin K1 for 6 weeks A 4-year-old male, neutered cat is presented with a history of severe lethargy and inappetance. The cat's mucous membranes are extremely pale and his PCV at presentation is 9%. [Normal=30-45%]. A blood transfusion is indicated. Which one of the following choices best explains the rationale for blood typing prior to transfusion in cats? A. Cats have naturally-occurring antibodies against the blood group antigen they lack B. Blood typing is not necessary. Cats are universal receivers and can receive any blood type C. Cats that are negative for DEA 1.1 are more likely to have a reaction to DEA 1.1 positive blood D. If donor is Type A, it is a universal donor and recipient typing is not needed E. Blood typing will determine if recipient cat is Rh factor negative or Rh positive - CORRECT ANSWER: A. Cats have naturally-occurring antibodies against the blood group antigen they lack A 4-year-old Quarter horse is presented with a 3-week history of diffuse patchy alopecia on the ventral midline and face, including a "bulls-eye" lesion in the center of the forehead. There is a mild conjunctivitis and uveitis OU (both eyes). A punch skin biopsy looks like the image below. What is the most appropriate treatment? A. Ivermectin now, repeat in 4 months B. Trichlorfon drench, antibiotic ophthalmic ointment TID C. Do nothing D. Corticosteriods, benzimidazole-piperazine q. 12 weeks E. Topical DMSO, Oxybendazole q. 8 weeks - CORRECT ANSWER: A. Ivermectin now, repeat in 4 months Diagnosis is Onchocerca A 4-year-old Standardbred mare is examined in the fall with a 5-day history of depression, left head tilt, facial paralysis and worsening gait. The horse is ataxic and knuckles on both forelegs, worse on the right. There is muscle atrophy of the left shoulder and right hindquarters and strips of localized spontaneous sweating over the left trunk. Which diagnostic test would best support the presumptive diagnosis? A. Serum:CSF antibody titer ratios for Sarcocystis neurona B. MRI to rule out nigropallidal encephalomalacia C. Serum IgM capture ELISA for West Nile Virus D. Serum Western blot analysis for eastern equine encephalomyelitis E. Plasma antibody test for St. Louis encephalitis - CORRECT ANSWER: A. Serum:CSF antibody titer ratios for Sarcocystis neurona A 5 y/o FS co*cker Spaniel is presented with a strange expression. Her right ear and lip appear to droop. There is ptosis OD and the dog is drooling on the exam table. What anatomic structure is damaged? A. Facial nerve B. Left side inner ear C. Trigeminal nerve D. Right side inner ear E. Left side medulla, motor tract - CORRECT ANSWER: A. Facial nerve A 5-month old kitten presents to you with a rectal prolapse, as shown in the photo. What is the most common cause of rectal prolapse in a kitten? A. Panleukopenia B. Gastrointestinal parasites C. Trauma D. Dysautonomia - CORRECT ANSWER: B. Gastrointestinal parasites A 5-month old kitten presents to you with a rectal prolapse. What is the most common cause of rectal prolapse in a kitten? A. Trauma B. Panleukopenia C. Gastrointestinal parasites D. Dysautonomia - CORRECT ANSWER: C. Gastrointestinal parasites A 5-year old DSH female cat presents with swollen edematous paws, facial lesions, and oral ulcerations and vesicles. The housemate died quickly 1 day ago. What is your most likely diagnosis? A. Virulent systemic disease (caused by feline calicivirus) B. Feline rhinotracheitis C. Bordetella bronchiseptica D. Feline coronavirus - CORRECT ANSWER: A. Virulent systemic disease (caused by feline calicivirus) A 5-year old indoor/outdoor male neutered short hair presents for a wound on the chest. The owner noticed a hole in the skin and thought he had a ruptured abscess. On close examination of the wound, you notice a larva inside the hole. Which organism is most likely under the skin? A. Ctenocephalides B. Cuterebra C. Ancylostoma D. Culicoides - CORRECT ANSWER: B. Cuterebra A 5-year old male castrated Mastiff presents for left pelvic limb lameness. The medial aspects of both stifles are thickened. Manipulation of the left stifle reveals cranial motion of the tibia relative to the femur and a clicking sound from the joint on flexion and extension. What is the most likely diagnosis? A. Left cranial cruciate ligament rupture with meniscal cartilage tear B. Left caudal cruciate ligament rupture with no meniscal cartilage tear C. Left luxating patella D. Left cranial cruciate ligament rupture with no meniscal cartilage tear - CORRECT ANSWER: A. Left cranial cruciate ligament rupture with meniscal cartilage tear A 5-year-old Arabian mare expels the placenta 2 hours after foaling. On examination, it appears that the placenta has passed "inside out," with the allantoic side of the allantochorion exposed. What is the correct interpretation? A. Normal finding B. Sign of placental insufficiency C. Suggests premature placental separation D. Do not rebreed this mare E. Foal at risk for peripartum asphyxia - CORRECT ANSWER: A. Normal finding A 5-year-old Arabian mare is presented. The horse was vaccinated against eastern and western equine encephalitis virus, West Nile virus, and strangles just two weeks ago. The horse has a 48-hour history of urticarial wheals on its skin that progressed to sharply demarcated edema of the muzzle, eyes, belly and limbs. She reacts as if it is painful when the edema is checked. The horse breathes heavily, with stridor, dyspnea and diarrhea. There are petechiation and ecchymoses on the mucous membranes and the muzzle is a reddish, mulberry color. What is the clinical diagnosis? Value Normal 102.2 F (39 C) 99.0-100.6, 37.2-38.1 C HR=32 bpm 28-40 bpm BR=12 brpm 10-14 brpm A. Potomac horse fever B. Equine salmonellosis C. Lyme disease D. Purpura hemorrhagica E. Equine viral arteritis (EVA) - CORRECT ANSWER: D. Purpura hemorrhagica A 5-year-old cat is being evaluated because of a sudden onset of paralysis of the rear legs. Femoral pulses are absent, and the rear legs are cold and painful. Spinal reflexes of the rear legs are absent. Which of the following is the most likely diagnosis? A. Aberrant parasitic migration B. Aortic thromboembolism C. Trauma to the spinal cord D. Intervertebral disk herniation - CORRECT ANSWER: B. Aortic thromboembolism A 5-year-old female spayed Australian shepherd is presented for poor appetite and lethargy. Bloodwork reveals hypercalcemia. Urinalysis is normal. Abdominal ultrasonography is unremarkable. Which one of the following choices is the most likely diagnosis? normochromic anemia with a PCV of 29% (35-57%), lipemic serum, and cholesterol of 1090 mg/dl (135-278 mg/dl). What is the most likely diagnosis? A. Adrenal dependent hyperadrenocorticism B. Pituitary dependent hyperadrenocorticism C. Hyperthyroidism D. Hypothyroidism - CORRECT ANSWER: D. Hypothyroidism A 6-year old female spayed indoor only domestic medium hair presents with a large raw lesion on its upper left lip. The owner reports the lesion has come and gone over the past few years. Which of these treatments would likely be most helpful? A. Topical trifluridine and systemic clindamycin B. Marginal surgical excision and radiation therapy C. Wide surgical excision and chemotherapy D. Immunosuppressive therapy and a change to a hypoallergenic diet - CORRECT ANSWER: D. Immunosuppressive therapy and a change to a hypoallergenic diet A 6-year old female spayed indoor only domestic medium hair presents with a large raw lesion on its upper left lip. The owner reports the lesion has come and gone over the past few years. Which of these treatments would likely be most helpful? A. Wide surgical excision and chemotherapy B. Immunosuppressive therapy and a change to a hypoallergenic diet C. Topical trifluridine and systemic clindamycin D. Marginal surgical excision and radiation therapy - CORRECT ANSWER: B. Immunosuppressive therapy and a change to a hypoallergenic diet A 6-year old Friesian cow presents to you with a mass on the left mandible (see image). The farmer reports that the mass has developed over the last several weeks and the cow has recently had some difficulty eating and lost weight. On examination, the mass is firm, immobile, and painful on manipulation. You note a thick discharge with small granular particles. Based on the most likely diagnosis, what should you tell the farmer? A. Treatment with sodium iodide intravenously is likely curative but the cow's milk and meat will not be suitable for human consumption for 120 days B. Intravenous penicillins are likely to be effective C. The most effective treatment is oral sodium iodide D. Treatment is unlikely to be successful and the cow should be culled - CORRECT ANSWER: D. Treatment is unlikely to be successful and the cow should be culled A 6-year old Friesian cow presents to you with a mass on the left mandible. The farmer reports that the mass has developed over the last several weeks and the cow has recently had some difficulty eating and lost weight. On examination, the mass is firm, immobile, and painful on manipulation. You note a thick discharge with small granular particles. Based on the most likely diagnosis, what should you tell the farmer? A. Treatment with sodium iodide intravenously is likely curative but the cow's milk and meat will not be suitable for human consumption for 120 days B. Intravenous penicillins are likely to be effective C. The most effective treatment is oral sodium iodide D. Treatment is unlikely to be successful and the cow should be culled - CORRECT ANSWER: D. Treatment is unlikely to be successful and the cow should be culled A 6-year old intact male domestic short haired cat presents with acute onset exophthalmos of the right eye in the past 2 days. He is painful on palpation around his eye and head. The eye can be retropulsed some, but there is resistance present. He has not been eating in the past day and has a rectal temperature of 103.4F (39.7 C). What is the most likely diagnosis? A. Retrobulbar abscess B. Anterior uveitis C. Orbital neoplasia D. Glaucoma - CORRECT ANSWER: A. Retrobulbar abscess A 6-year old male neutered domestic shorthair cat is presented with 4-month history of weight loss and regurgitating undigested food immediately after eating. Which one of the following abnormalities is apparent on the patient's radiographs? A. Megaesophagus B. Pulmonary mass C. Left atrial enlargement D. Pneumonia E. Diskospondylitis - CORRECT ANSWER: A. Megaesophagus A 6-year old male neutered terrier mix weighing 10kg presents to you after having ingested 3 pieces of sugar-free gum containing xylitol. What bloodwork abnormality are you expecting to find? A. Hypoglycemia B. Hyperglycemia C. Hypernatremia D. Hyponatremia E. Hypocalcemia - CORRECT ANSWER: A. Hypoglycemia A 6-year old male neutered terrier mix weighing 10kg presents to you after having ingested 3 pieces of sugar-free gum containing xylitol. What bloodwork abnormality are you expecting to find? A. Hypoglycemia B. Hypernatremia C. Hyperglycemia D. Hypocalcemia E. Hyponatremia - CORRECT ANSWER: A. Hypoglycemia A 6-year old male neutered Weimaraner presents for left forelimb lameness. Radiographs are shown below and show a mixed productive and destructive lesion affecting the left distal radius with accompanying soft tissue swelling. The lesion does not cross the joint. What is the most common primary bone tumor in the dog? A. Multiple myeloma B. Fibrosarcoma C. Chondrosarcoma D. Osteosarcoma E. Hemangiosarcoma - CORRECT ANSWER: D. Osteosarcoma A 6-year old Standardbred jumper is presented with a 3-month history of poor performance and intermittent shifting hindleg lameness. On physical exam, there is poor muscling of the gluteal muscles and some asymmetry to the croup (rump). The horse has a 7 cm. swelling over the hindquarters on the left dorsal side and shows pain and a reluctance to ventroflex the back when midline pressure is applied. On rectal palpation crepitation and shifting can be felt dorsally as the horse walks slowly forward. What is the diagnosis? A. Croup myopathy (longissimus dorsi, supraspinous ligament) B. Coxofemoral luxation C. Sacroiliac subluxation D. Overlapping vertebral spinous processes E. Fibrotic ossifying myopathy - CORRECT ANSWER: C. Sacroiliac subluxation A 6-year old Sun Conure presents for evaluation of a skin lesion. The owner reports that the bird is very sensitive on the wing near the lesion. On your exam, you note an oval swelling involving the feather follicle as seen in the image below. What is the best treatment for feather cysts? A. Treat with systemic antibiotics for 4-6 weeks B. Squeeze and express the material out of the feather cyst C. Lance and drain the feather cyst, flush with saline D. Surgical removal of the feather cyst and follicle E. No treatment is necessary as feather cysts typically resolve spontaneously - CORRECT ANSWER: D. Surgical removal of the feather cyst and follicle A 6-year old Sun Conure presents for evaluation of a skin lesion. The owner reports that the bird is very sensitive on the wing near the lesion. On your exam, you note an oval swelling involving the feather follicle. What is the best treatment for feather cysts? A. Surgical removal of the feather cyst and follicle B. Lance and drain the feather cyst, flush with saline C. Squeeze and express the material out of the feather cyst D. Treat with systemic antibiotics for 4-6 weeks E. No treatment is necessary as feather cysts typically resolve spontaneously - CORRECT ANSWER: A. Surgical removal of the feather cyst and follicle A 6-year old West Highland White Terrier comes in to see you for the mucopurulent ocular discharge as seen in the photo below. A Schirmer tear test shows no tear production. What is the treatment of choice for chronic canine keratoconjunctivitis sicca? A. Topical cyclosporine and a topical steroid B. Systemic antibiotics and corticosteroids C. Systemic cyclosporine and antibiotics A 7-year old female spayed Border Collie is presented with two very goopy, gunk- covered eyes. A Schirmer tear test reveals less than 10 mm/minute of wetting [N= 15mm or more/ minute]. The owner reports that the dog has been on "some kind of medicine" for the last 10 days, but it is not his dog, and he doesn't know what the medicine is. Keratoconjunctivitis sicca (KCS) secondary to the drug is suspected. Which one of the following drugs may be causing the KCS? A. Amitraz B. Trimethoprim-sulfa C. Griseofulvin D. Itraconazole E. Prednisolone - CORRECT ANSWER: B. Trimethoprim-sulfa A 7-year old female spayed English Springer Spaniel presents for difficulty breathing and lethargy. Physical exam shows pale mucous membranes and increased respiratory effort. You collect blood for a complete blood count and a chemistry panel and you place a drop of blood with a drop of saline on a glass slide. After swirling the sample around, you can grossly see what is shown in the picture. What's your diagnosis? A. Rouleaux formations B. Evan's syndrome C. Hemolysis D. Immune mediated hemolytic anemia E. Vitamin K antagonist toxicity - CORRECT ANSWER: D. Immune mediated hemolytic anemia A 7-year old female spayed Standard Poodle presents with weakness and lethargy. A chemistry panel shows a Na+ = 130 mEq/L (142-152 mEq/L), K+ = 6.5 mEq/L (3.9-5.1 mEq/L), BUN 55 mg/dl (8-28 mg/dl), creatinine 1.9 mg/dl (0.5-1.7 mg/dl). The test to run for a definitive diagnosis would be which of the following? A. ACTH stimulation test B. Thoracic radiographs C. Electrocardiogram D. Bile acids test - CORRECT ANSWER: A. ACTH stimulation test A 7-year old male castrated cat presents for respiratory distress with open-mouth breathing. You initially place him in oxygen and obtain thoracic radiographs when he is stabilized. The films are shown here. What is your diagnosis? A. Left-sided congestive heart failure B. Neoplasia C. Pneumonia D. Feline asthma syndrome E. Tracheal collapse - CORRECT ANSWER: A. Left-sided congestive heart failure A 7-year old male castrated cat presents to you for difficulty eating. On examination, you see that there are dental lesions on the buccal surfaces of several premolar and first molar teeth. The cat shows signs of discomfort when you palpate around these teeth and the surrounding gingiva appears inflamed. You suspect that the cat has odontoclastic resorptive lesions. You perform dental radiographs which show evidence of endodontic necrosis. Which of the following is the most appropriate treatment recommendation? A. Affected teeth should be extracted and it is likely that other teeth will be affected in the future B. Affected teeth should be extracted and Vitamin D supplementation may reduce the likelihood of development of similar lesions in other teeth C. Administration of an analgesic may provide relief until the lesion spontaneously resolves D. Affected teeth should be treated by removal of the crown and coron - CORRECT ANSWER: A. Affected teeth should be extracted and it is likely that other teeth will be affected in the future A 7-year old male German shepherd is presented with a history of weakness in the hind limbs, urinary incontinence and recent obsessive chewing around his tail area. Dorsiflexion of the tail over the back and extension of the hind limbs elicits a painful response. There is moderate hindlimb ataxia. He does not withdraw each hind leg when a toe is pinched, but bears weight on the hindlimbs. Patellar reflexes are normal. What is the clinical diagnosis? A. Wobbler syndrome B. Diskospondylitis C. Hip dysplasia D. Cauda equina syndrome E. Radiculoneuritis - CORRECT ANSWER: D. Cauda equina syndrome A 7-year old male intact Chesapeake Bay Retriever presents to your clinic with the presenting complaint of an intermittent cough. On exam, the dog is bright and alert with a temperature of 100.5F (38.1 C), heart rate of 110 beats per minute and respiratory rate of 30 breaths per minute. You perform chest radiographs which are shown below. A CBC shows a hematocrit of 39% (35-57%), neutrophil count of 8,659/ul (2,900- 12,000/ul), monocyte count of 984/ul (100-1,400/ul) and eosinophil count of 1,980/ul (0- 1,300/ul). What is the treatment of choice for the most likely diagnosis? A. Pericardiocentesis B. Immiticide (Melarsomine) C. Furosemide D. Enrofloxacin E. Terbutaline - CORRECT ANSWER: B. Immiticide (Melarsomine) Radiograph showed prominent pulmonary arteries with right ventricular enlargement, classic of HWD A 7-year-old cat is presented with miosis, ptosis, enophthalmos, and protruded nictitating membrane involving the left eye. These symptoms are caused by DAMAGE to which one of the following innervation pathways? A. Right trigeminal innervation to the eye B. Right parasympathetic innervation to the eye C. Left oculomotor innervation to the eye D. Left sympathetic innervation to the eye E. Right vagal innervation to the eye - CORRECT ANSWER: D. Left sympathetic innervation to the eye A 7-year-old Thoroughbred event gelding is presented after coming up lame on the left forelimb following completion of a course. The horse is 4/5 lame on the left forelimb. There is moderate swelling over the palmar aspect of the left metacarpus as seen in the image below. The horse is sensitive to palpation of the swollen area. Which diagnostic would be most appropriate to recommend? A. Ultrasound the metacarpus B. Digital radiographs of the distal limb C. Nuclear scintigraphy (a.k.a. bone scan) D. Diagnostic perineurial anesthesia (a.k.a. nerve blocks) E. No further diagnostics needed - diagnosis is clear - CORRECT ANSWER: A. Ultrasound the metacarpus A 9-month old male pitbull presents to you for hindlimb lameness that developed suddenly several days ago and has gotten worse. On your examination, the dog is painful on palpation of the left tibia. You take radiographs of the tibia which are shown below. Which of the following is the most appropriate treatment for the suspected condition? A. Splint stabilization B. Surgical correction C. Carprofen D. Ampicillin E. Fluconazole - CORRECT ANSWER: C. Carprofen A 9-year old male Queensland Heeler presents with a four day history of progressive tetraparesis. Physical exam showed him to be weakly ambulatory with support. As part of your initial workup, you take chest X-rays which are shown below. Which of the following next steps is the most appropriate test to confirm your clinical suspicion about the cause of the dog's signs? A. Tensilon response test B. MRI of the brain C. Myelogram D. Bronchoalveolar lavage and culture E. CT scan of the thorax - CORRECT ANSWER: A. Tensilon response test Chest X-rays indicated the presence of a thymoma, which would cause the clinical signs of secondary myasthenia gravis. A 9-year-old German shepherd is presented with unchecked bleeding from a cut on the gums above the right canine tooth. The owner relates that the dog has lost weight and had an episode of collapse 3 days ago, but he recovered. On physical exam, the gums are pale with petechiae and ecchymotic hemorrhages. There is tachycardia and a palpable cranial abdominal mass. A coagulation profile shows the following: A 10-year old male castrated cat that you have previously diagnosed with hyperthyroidism presents to you for acute onset of blindness. You perform an ophthalmic exam and note retinal hemorrhage. What diagnostic test should you perform first? A. Blood pressure B. Total T4 levels C. Serum BUN and creatinine D. Free T4 levels by equilibrium dialysis E. Coagulation times - CORRECT ANSWER: A. Blood pressure A 10-year old male castrated cat that you have previously diagnosed with hyperthyroidism presents to you for acute onset of blindness. You perform an ophthalmic exam and note retinal hemorrhage. What diagnostic test should you perform first? A. Serum BUN and creatinine B. Coagulation times C. Total T4 levels D. Free T4 levels by equilibrium dialysis E. Blood pressure - CORRECT ANSWER: E. Blood pressure A 10-year old neutered male cat is presented with vague signs of anorexia, occasional vomiting and lethargy. On physical examination, the patient is somewhat dehydrated. Value Normal T=99.1 F (37.3 C) 100-103.1 F, 37.8-39.5 C Bloodwork Value Normal ALB=2.2 g/dL 2.6-4.0 g/dL ALK PHOS=88 U/L 3-65 U/L ALT=112 U/L 8.2-57 U/L AMYL=2483 U/L 270-1,462 U/L BUN=42 mg/dL 10-30 mg/dL K=3.3 3.5-5.1 CL=108 mEq/L 109-122 mEq/L WBC=22,200 3800-19,500 LYMPHS=1404/uL 1000-4800/uL NEUTS=15538/uL 0-3000/uL Trypsin-like immunoreactivity 149.0 µg/L 12.0 - 82.0 µg/L Which one of the following diseases is most likely, based on these findings? A. Pancreatitis B. Feline infectious peritonitis C. Hepatic lipidosis D. Cholelithiasis E. Toxoplasma gondii - CORRECT ANSWER: A. Pancreatitis A 10-year-old male neutered DSH cat is presented with a history of being hit by a car one month ago. There is a recent onset of weight loss. Lung sounds are muffled on the right side. What is the most likely diagnosis based on interpretation of these radiographs? A. Pleural effusion B. Aspiration pneumonia C. Vascular ring anomaly D. Megaesophagus E. Diaphragmatic hernia - CORRECT ANSWER: E. Diaphragmatic hernia Radiographs showed soft tissue opacity in the right hemithorax, with displacement of the trachea and carina A 10-year-old male Pomeranian presents to you with a 3-month history of wheezing and marked respiratory distress. The owner reports that he has been coughing sporadically as if trying to clear his throat of a hairball. The owner is a regular smoker. On your examination, the dog displays moderate expiratory difficulty. You submit blood work and take radiographs including inspiratory and expiratory lateral projections of the dog, which are shown below. Which of the following are commonly used to treat the likely condition? A. Meloxicam and fluconazole B. Surgical repair of the defect C. Amoxicillin, nebulization, and coupage D. Butorphanol and theophylline E. Furosemide and prednisone - CORRECT ANSWER: D. Butorphanol and theophylline A 10-year-old spayed female cat is being evaluated because of weight loss, polyphagia, polyuria, polydipsia, and restlessness. Cardiac arrhythmias are noted on auscultation. Which of the following is the most likely diagnosis? A. Chronic pancreatic insufficiency B. Chronic renal disease C. Congestive cardiomyopathy D. Diabetes mellitus E. Hyperthyroidism - CORRECT ANSWER: E. Hyperthyroidism A 11-year old male castrated Golden Retriever presents for collapse with muffled heart sounds on examination. You ultrasound the heart and obtain the following image; the right atrium (RA) and right ventricle (RV) are labeled. A large mass is seen in the right atrioventricular groove with pericardial effusion. What is the likely diagnosis? A. Pulmonary adenocarcinoma B. Chemodectoma C. Hemangiosarcoma D. Idiopathic pericardial hemorrhage E. Lymphoma - CORRECT ANSWER: C. Hemangiosarcoma A 12 year old neutered male black labrador retriever is presented with a 3 week history of limping on the right fore. The lateral digit is swollen and the nail is deviated ventrally, with ulceration of the nail bed. A lytic bone lesion of the 2nd phalanx is visible on a radiograph and cytology of the mass suggests neoplasia. Chest radiographs are clear. Following amputation of the digit, histopathology indicates that the mass is a squamous cell carcinoma. Which of the following is the best advice for the owner? A. 95% chance he will survive 1 year B. Chemotherapy is indicated C. Radiotherapy is indicated D. Guarded prognosis E. 50% chance he will survive 1 year - CORRECT ANSWER: A. 95% chance he will survive 1 year A 12- year-old spayed female black Scottish terrier is presented with a three week history of limping on the right forelimb. The second digit of the paw is enlarged, ulcerated, and bleeding with a loose toenail. A lytic bone lesion of the 3rd phalanx is visible on radiographs, and cytology of a fine needle aspirate of the mass suggests neoplasia. If biopsy of the mass confirms malignant subungual melanoma, which one of the following choices contain the best advice to the owner? A. Long-term prognosis is excellent if the tumor is completely resected. B. Metastasis is common. Radical excision, plus adjuvant therapies is typically recommended C. Due to the exquisite sensitivity of melanoma to radiotherapy, it is preferred as a sole treatment D. DNA-based vaccine encoding human tyrosinase is effective against oral tumors but not licensed for subungual melanoma E. Nail bed melanomas in dogs are almost always benign and re - CORRECT ANSWER: B. Metastasis is common. Radical excision, plus adjuvant therapies is typically recommended. A 12-year old cat presents with an abnormal gait and appears to be walking with the both hocks dropped down low to the ground. The owner says the cat has been losing weight and drinking a lot of water. What diagnostic test should you run? A. Blood glucose B. Echocardiogram C. BUN and Creatinine D. MRI of the brain - CORRECT ANSWER: A. Blood glucose A 12-year old male neutered cat weighing 14 pounds is presented with a 2-month history of PU/PD, increased appetite, lameness, weight gain, exercise intolerance and dyspnea. Physical exam shows a systolic heart murmur with a gallop rhythm, a lateral chest radiograph shows pulmonary edema and a large heart. Which one of the following choices is the most likely diagnosis? Value Normal T=102.0 F (38.9 C) 100-103.1 F, 37.8-39.5 C HR=110 bpm 120-140 bpm BR=24 brpm 6-40 brpm B. Right cerebrum C. Left brainstem D. Left cerebrum E. Cerebellum - CORRECT ANSWER: D. Left cerebrum A 16-year old budgerigar presents with progressive unilateral paresis of the right leg (see image). There is normal flexion and extension of the hip joint with decreased flexion, extension, and sensation below the knee. The remainder of your skeletal and neurologic examination is unremarkable. Which of the following is most likely in this bird? A. Renal neoplasia B. Botulism C. Marek's disease D. Lead toxicosis E. Intervertebral disc disease - CORRECT ANSWER: A. Renal neoplasia -Pressure exerted on the sciatic nerve by a renal tumor A 16-year old budgerigar presents with progressive unilateral paresis of the right leg. There is normal flexion and extension of the hip joint with decreased flexion, extension, and sensation below the knee. The remainder of your skeletal and neurologic examination is unremarkable. Which of the following is most likely in this bird? A. Intervertebral disc disease B. Lead toxicosis C. Botulism D. Renal neoplasia E. Marek's disease - CORRECT ANSWER: D. Renal neoplasia A 21-year old cat with a history of hyperthyroidism, renal insufficiency, a heart murmur, recurrent urinary tract infections, and inflammatory bowel disease presents to you for a second opinion about the cat's facial excoriations and severe pruritus. You ask the owner about the cat's medications and she is unsure exactly what the cat has been receiving for those ailments. Which of the following medications is most likely responsible for the cat's signs? A. Metronidazole B. Methimazole C. Enrofloxacin D. Metoclopramide - CORRECT ANSWER: B. Methimazole A 21-year-old scarlet macaw is presented with polydipsia, watery droppings and inappetence. On examination, the bird was fluffed and depressed with minimal pectoral muscle atrophy. The crop was fluctuant. Which one of the following choices is the most likely diagnosis? A. Thymoma B. Air sacculitis C. Vitamin D toxicosis D. Retained egg E. Normal radiographs - CORRECT ANSWER: D. Retained egg A 36 hour foal born from a multiparous mare presents for weakness, decreased appetite, and tachycardia. On physical exam, you examine the sclera as seen in the photo. Based on the signalment and findings, what is a likely cause? A. Congenital iron toxicity B. Immune mediated thrombocytopenia C. Neonatal Isoerythrolysis D. Failure of passive transfer - CORRECT ANSWER: C. Neonatal isoerythrolysis (sclera in photo were icteric) A bovine diet that is low in thiamine or high in sulfur can cause ____________. A. Polioencephalomalacia B. Pregnancy toxemia C. Parturient paresis D. Downer cows E. Pseudorabies - CORRECT ANSWER: A. Polioencephalomalacia A budgerigar is showing signs of squamous metaplasia of the oral mucosa, conjunctiva, and upper airways. It has developed associated bacterial sinusitis. In a pet bird, what are these clinical signs most suggestive of? A. Hypocalcemia B. Iodine deficiency C. Hypervitaminosis D D. Hypovitaminosis D E. Hypovitaminosis A - CORRECT ANSWER: E. Hypovitaminosis A A canine coagulation profile indicates increased values for APTT, PT, TT and FDPs. Thrombocytes = 62,513 per microliter (normally greater than 211,000 per microliter). What is the typical prognosis for a dog with the condition suggested by this pattern? A. Good B. Fair C. Grave D. Excellent E. Need more information - CORRECT ANSWER: C. Grave -These values are suggestive of DIC --> "Death Is Coming" A cat presents to you dyspneic and obtunded after falling from a tall tree. On physical examination, he is painful on manipulation of the jaw. You auscult decreased lung sounds dorsally. What should you do first? A. Chest radiograph B. Place the cat in an oxygen cage C. Thoracocentesis D. Skull radiograph - CORRECT ANSWER: C. Thoracocentesis A cat with a previous diagnosis of diabetes mellitus confirmed by persistent fasting hyperglycemia and persistent glycosuria is presented for a routine check-up. The owner relates that she ran out of injectable insulin two months ago, but the cat seemed to do fine without it, so she stopped giving insulin shots. On physical exam, the cat appears healthy and a dipstick test shows a blood glucose level of 125 mg/dL (6.9 mmol/l) [normal 61-132 mg/dL (3.5-7.3 mmol/l)]. What is the most likely explanation? A. Type I diabetes mellitus B. Concurrent diabetes insipidus C. Incorrect initial diagnosis D. Diabetic remission E. Insulin resistance - CORRECT ANSWER: D. Diabetic remission A chicken farm has had several ill and dead birds over the past week. Examination of the sick chickens shows severe depression, ecchymoses and edema of the comb and wattles, green diarrhea. Some are unable to walk with absent perching reflexes. Which one of the following choices is the most likely diagnosis? A. Avian encephalomyelitis B. Avian influenza C. Infectious coryza D. Infectious bronchitis E. Mycoplasma gallisepticum - CORRECT ANSWER: B. Avian influenza A chicken operation has recently been ravaged by a respiratory disease affecting almost all of the chickens in the flock. The chickens are coughing and sneezing and many have facial swelling. You necropsied many of the chickens and found mucoid exudate in the bronchi, thickened air sacs, and in a few of the chickens, interstitial nephritis was present. Which of these diseases is likely? A. Aspergillus B. Infectious bursal disease C. Fowl cholera D. Infectious bronchitis - CORRECT ANSWER: D. Infectious bronchitis A chicken operation has recently been ravaged by a respiratory disease affecting almost all of the chickens in the flock. The chickens are coughing and sneezing and many have facial swelling. You necropsied many of the chickens and found mucoid exudate in the bronchi, thickened air sacs, and in a few of the chickens, interstitial nephritis was present. Which of these diseases is likely? A. Infectious bronchitis B. Infectious bursal disease C. Aspergillus D. Fowl cholera - CORRECT ANSWER: A. Infectious bronchitis A client brings the one-half inch grub shown in the image to you one spring day, telling you it emerged from a hole in the back of one of his prize show cattle. He wants to know what to treat his cattle with and when. A. Ivermectin in early fall responsible for transmitting the infectious agent most likely responsible for this goat's condition? A. Amblyomma spp. ticks B. Dermacentor spp. ticks C. Damalinia caprae lice D. Musca autumnalis (the face fly) E. Tabanids (e.g., the horse fly) - CORRECT ANSWER: A. Amblyomma spp. ticks A dog presents to your clinic with tenesmus and swelling near the anus as seen in the image. On examination, there is a fluctuant swelling lateral to the anus, and on rectal exam, you note lateral dilatation of the rectum. Which of the following is the most common signalment for dogs presenting with this problem? A. There is no age or sex predilection B. They are younger intact females C. They are younger intact males D. They are older intact females E. They are older intact males - CORRECT ANSWER: E. They are older intact males A dog presents to your clinic with tenesmus and swelling near the anus. On examination, there is a fluctuant swelling lateral to the anus, and on rectal exam, you note lateral dilatation of the rectum. Which of the following is the most common signalment for dogs presenting with this problem? A. They are younger intact females B. They are older intact males C. There is no age or sex predilection D. They are younger intact males E. They are older intact females - CORRECT ANSWER: B. They are older intact males A dog presents with acute onset vomiting, hemorrhagic diarrhea and fever. On fecal examination, you find many large fluke eggs. You question the owner and discover that the dog was recently in Oregon on a boating trip. What agent is most likely causing the clinical signs in this dog? A. Neorickettsia helminthoeca B. Oxytrema silicula C. Nanophyetus salmincola D. Rickettsia rickettsii - CORRECT ANSWER: A. Neorickettsia helminthoeca A farmer comes to you with a new test for detection of mastitis in his heifers. He claims that both the sensitivity and specificity of his test is 90%. Mastitis has a prevalence of 5% in his herd. The negative predictive value (NPV) is approximately _________. A. 99% B. 35% C. 72% D. 3% - CORRECT ANSWER: A. 99% A farmer recently purchased a ram from a feedlot. The ram soon became ill, with progressive depression, lethargy, and inappetance. He had respiratory problems, breathing hard with abdominal effort, and died after three days with a frothy nasal discharge just prior to death. At necropsy, the lungs had the appearance shown below. What was the most likely cause of death? A. Ovine progressive pneumonia B. Mannheimia haemolytica C. Corynebacterium pseudotuberculosis D. Dictyocaulus filaria E. Perilla frutescens - CORRECT ANSWER: B. Mannheimia haemolytica A female show dog is approaching estrus and the owner has consulted you about planning a breeding for this dog. She is aware of the signs of proestrus and plans to contact you when those are apparent in her dog. In addition to vagin*l cytology, which one of the following will be the most useful to you in planning the timing of breeding? A. Vaginoscopy B. Determination of serum estrogen levels C. Timed administration of luteinizing hormone D. Serial progesterone testing E. Ovarian ultrasonography - CORRECT ANSWER: D. Serial progesterone testing A five-month-old Yorkshire terrier is presented with ataxia in all four limbs and neck pain. There is loss of conscious proprioception in all four limbs. A radiograph of the cervical spine is shown below. Which one of the following choices is the most likely diagnosis? A. Atlantoaxial instability B. Caudal occipital malformation C. Intervertebral disk disease D. Discospondylitis E. Spina bifida - CORRECT ANSWER: A. Atlantoaxial instability A five-year old lactating Holstein dairy cow is being evaluated for a two-month history of watery diarrhea, weight loss, and decline in milk production. On examination, the cow is bright, alert and responsive, but unthrifty and very thin, with a soft intermandibular swelling. What is the most appropriate management of this herd? A. Cull positives, ensure 4 L. colostrum uptake for calves B. Quarantine herd, treat until herd tests negative C. Test herd, treat positive animals, add coccidiostat to ration D. Test herd, cull positives, remove calves from dam at birth E. Test herd, treat positive animals, test and treat replacements - CORRECT ANSWER: D. Test herd, cull positives, remove calves from dam at birth A flock from a turkey farm is presented with a mysterious illness. Several dead birds are notable, mostly younger animals. Sick turkeys are listless, with drooping wings, unkempt feathers, yellow droppings. Sick older birds are emaciated. Necropsy shows a yellowish green, caseous exudate in the ceca, cecal ulcerations and thickening of the cecal wall. What is the diagnosis? A. Necrotic enteritis B. Histomoniasis C. Avian spirochetosis D. Coronaviral enteritis of turkeys E. Hemorrhagic enteritis of turkeys - CORRECT ANSWER: B. Histomoniasis A flock of chickens from a poultry operation is presented with an outbreak which began two days ago with the unexpected discovery of 20 dead birds. Since then, illness has affected about 25% of the flock. Birds are depressed and anorexic, many with mucoid discharge from the beak, ruffled feathers, diarrhea and increased respiratory rate. Two birds have torticollis, and several chickens with swollen sternal bursae, wattles, joints, tendon sheaths, and footpads. What is the clinical diagnosis? A. Infectious bronchitis B. Fowl cholera C. Infectious coryza D. Infectious laryngotracheitis E. Newcastle disease - CORRECT ANSWER: B. Fowl cholera A flock of sheep has been grazing on Sorghum halepense (Johnson grass), Tetradymia spp. (horsebrush) and Artemisia nova (black sage). Many of the exposed animals have edema, especially in their heads. This lamb has edema around the eyes and the ears show hyperkeratinized tissue. What is the photosensitizing agent responsible for this condition? A. Tetradymia spp. B. Chlorophyll C. Phylloerythrin D. Artemisia nova E. Bile acids - CORRECT ANSWER: C. Phylloerythrin A four-month-old filly is presented with a four-day history of lethargy, heavy breathing, cough and decreased appetite. Value Normal 104.9 F (40.5 C) 99.0-100.6, 37.238.1 C HR=72 bpm 28-40 bpm R=44 brpm 10-14 brpm On lung auscultation asymmetrically-distributed crackles and wheezes are audible. Some areas have no breath sounds and a dull resonance on thoracic percussion. A lateral chest radiograph shows consolidated nodular lung lesions and mediastinal lymphadenopathy. Pneumonia due to Rhodococcus equi infection is confirmed based on culture and polymerase chain reaction results on a transtracheal wash. Which one of following choices is the most appropriate treatment? A. Clarithromycin and rifampin B. Chloramphenicol and aminophylline C. Vancomycin and atropine (if bronchospasm is observed) E. Ulnar nerve paralysis - CORRECT ANSWER: A. Fractured P3 A horse is suspected of having the muscle disease rhabdomyolysis, and you are seeking a laboratory test to help confirm the diagnosis. Of the following, which test would likely be the most helpful? A. Gamma glutamyl transferase (GGT) B. Creatine kinase (CK) C. Anion gap D. Ionized phosphate E. Sorbitol dehydrogenase (SDH) - CORRECT ANSWER: B. Creatine kinase (CK) A horse is suspected of having the muscle disease rhabdomyolysis, and you are seeking a laboratory test to help confirm the diagnosis. Of the following, which test would likely be the most helpful? A. Sorbitol dehydrogenase (SDH) B. Creatine kinase (CK) C. Anion gap D. Gamma glutamyl transferase (GGT) E. Ionized phosphate - CORRECT ANSWER: B. Creatine kinase (CK) A horse presents to you for chronic, recurrent laminitis and skin disease. You notice on your exam that the horse has a particularly thick, long, wavy, and matted coat. The owner mentioned that this developed many months ago. What is a likely diagnosis? A. Pheochromocytoma B. Cushing's disease (Pituitary Pars Intermedia Dysfunction) C. Hypothyroidism D. Diabetes insipidus - CORRECT ANSWER: B. Cushing's disease (Pituitary Pars Intermedia Dysfunction) A horse presents to you for chronic, recurrent laminitis and skin disease. You notice on your exam that the horse has a particularly thick, long, wavy, and matted coat. The owner mentioned that this developed many months ago. What is a likely diagnosis? A. Pheochromocytoma B. Hypothyroidism C. Cushing's disease (Pituitary Pars Intermedia Dysfunction) D. Diabetes insipidus - CORRECT ANSWER: C. Cushing's disease (Pituitary Pars Intermedia Dysfunction) A horse presents to you with a corneal ulcer. You are concerned because it appears to be infected, as shown in this image. You perform cytology and find gram negative rods. What is the most likely organism infecting the corneal ulcer in this horse? A. Staphylococcus spp. B. Pasteurella multocida C. Pseudomonas D. E. coli - CORRECT ANSWER: C. Pseudomonas A horse presents to you with a corneal ulcer. You are concerned because it appears to be infected. You perform cytology and find gram negative rods. What is the most likely organism infecting the corneal ulcer in this horse? A. Pseudomonas B. E. coli C. Pasteurella multocida D. Staphylococcus spp. - CORRECT ANSWER: A. Pseudomonas A horse presents to your clinic after ingesting a large amount of grain. What is your major concern? A. Acidosis B. Laminitis C. Impaction D. Choke E. Torsion - CORRECT ANSWER: B. Laminitis A horse recently imported from Dubai died after a being ill for about 2 weeks in a quarantine facility. He was febrile and very depressed, with dyspnea and respiratory distress in the last 24 hours. Other signs included hyperemia of the conjunctiva and edema all over the head and neck, especially the eyelids and the supraorbital fossae. Necropsy reveals a clear pericardial effusion, froth in the upper airways, and heavy distended lungs. Which one of the following choices is the most likely diagnosis? A. Equine influenza B. Equine infectious anemia C. African horse sickness D. Equine viral arteritis E. Piroplasmosis - CORRECT ANSWER: C. African horse sickness A kitten comes into your clinic with tremors, and it cannot regulate its movement well. You suspect it has cerebellar hypoplasia. Which of these diseases would be on your differential list? A. Feline Immunodeficiency Virus (FIV) B. Feline Leukemia Virus (FeLV) C. Feline Panleukopenia Virus D. Pseudorabies - CORRECT ANSWER: C. Feline Panleukopenia Virus A kitten presents with watery diarrhea. On fecal examination, a wet mount shows motile protozoal trophozoites and a centrifugal zinc sulfate fecal flotation shows cysts of the parasite. Which of the following is the most acceptable treatment? A. Praziquantel B. Albendazole C. Metronidazole D. Pyrantel - CORRECT ANSWER: C. Metronidazole (Note - Dr. Starkey says fenbendazole is quite effective at killing this parasite; perhaps moreso than this correct answer) A large male guinea pig from a group of four is presented with swollen and scabby hind feet from which Staphylococcus aureus is cultured. Which one of the following recommendations is the most appropriate choice? A. Isolate affected animal, treat with oral amoxicillin B. Improve sanitation, install smooth-floored enclosure C. Cull affected animal, increase ventilation of environment for remaining animals D. Tetracycline-medicated water for all animals E. Debridement and topical 1% butenafine cream - CORRECT ANSWER: B. Improve sanitation, install smooth-floored enclosure A lion is presented with a tooth fracture of an upper left premolar, exposing the pulp. Dental radiographs are normal and no periodontal disease is present. Which one of the following choices is the best treatment? A. Wait until the exposure fills in then re-radiograph B. Extraction C. Do nothing D. Root canal therapy E. Restore the tooth with light cured resin - CORRECT ANSWER: D. Root canal therapy Treatment needed due to pulp exposure, but since radiographs are normal and no periodontal disease is present, extraction is unnecessary A long eared owl with a history of wing amputation is presented with anorexia, lethargy, and ataxia. Based on the images shown below, which one of the following structures has pathology? A. Cloaca B. Gizzard C. Coelom D. Ventriculus E. Proventriculus - CORRECT ANSWER: C. Coelom A male goat presents for vocalizing and straining to urinate. He has been kicking at his abdomen. You detect crystals adherent to the hairs around the prepuce. What condition should you suspect and try to rule out first? A. Lower intestinal obstruction B. Acute severe pyelonephritis C. Urinary tract obstruction D. Upper intestinal obstruction - CORRECT ANSWER: C. Urinary tract obstruction A male goat presents for vocalizing and straining to urinate. He has been kicking at his abdomen. You detect crystals adherent to the hairs around the prepuce. What condition should you suspect and try to rule out first? A. Lower intestinal obstruction B. Urinary tract obstruction C. Acute severe pyelonephritis D. Upper intestinal obstruction - CORRECT ANSWER: B. Urinary tract obstruction A. Herpesviral hepatitis B. Theiler's disease C. Clostridium novyi type B D. Tyzzer's disease - CORRECT ANSWER: D. Tyzzer's disease A one-month old foal develops fever, icterus, and diarrhea acutely. Bloodwork shows hyperfibrinogenemia, hypoglycemia, and elevated liver enzymes. Which of these conditions is most likely? A. Herpesviral hepatitis B. Tyzzer's disease C. Clostridium novyi type B D. Theiler's disease - CORRECT ANSWER: B. Tyzzer's disease A one-year-old intact male domestic short hair cat is presented with dyspnea after vehicular trauma. On physical examination, the cat is open-mouth breathing with muffled heart sounds and borborygmi heard on pulmonary auscultation. Which one of the following choices is the most likely diagnosis? A. Cardiac tamponade B. Diaphragmatic hernia C. Posterior lung lobe torsion D. Flail chest E. Pneumothorax - CORRECT ANSWER: B. Diaphragmatic hernia A Paint horse mare gives birth to an all white foal (see image). What clinical sign would you expect to see in a foal with lethal white syndrome? A. Ataxia B. Anhydrosis C. Regurgitation D. Constipation - CORRECT ANSWER: D. Constipation A Paint horse mare gives birth to an all white foal. What clinical sign would you expect to see in a foal with lethal white syndrome? A. Constipation B. Anhydrosis C. Ataxia D. Regurgitation - CORRECT ANSWER: A. Constipation A petting zoo is experiencing a high mortality outbreak among young chickens, guinea fowl, pheasants and turkeys. Affected birds huddle near heat sources. They are off feed and weak, with whitish feces pasted around their vents (diarrhea). On necropsy, lesions include unabsorbed yolk sacs and gray nodules in the liver, spleen, lungs, heart, gizzard, and intestine. There are firm, cheesy cecal cores. This presentation is highly suggestive of which diagnosis? A. Campylobacter jejuni B. Pullorum disease C. Avian spirochetosis D. Histomoniasis E. Coronaviral enteritis - CORRECT ANSWER: B. Pullorum disease A pig farm has several sick and dying adult pigs. Affected animals are febrile and depressed. Some seem constipated and others have diarrhea. A few are ataxic. A necropsy on one of the dead pigs shows widespread petechial and ecchymotic hemorrhages in the kidneys ("turkey egg kidneys"), bladder, spleen, and larynx. Which one of the following choices is the most likely diagnosis? A. Haemophilus parasuis B. Hemagglutinating encephalomyelitis C. Classical swine fever D. Postweaning multisystemic wasting syndrome E. Anthrax - CORRECT ANSWER: C. Classical swine fever A pig farm has several sick and dying adult pigs. Affected animals are febrile and depressed. Some seem constipated and others have diarrhea. A few are ataxic. A necropsy on one of the dead pigs shows widespread petechial and ecchymotic hemorrhages in the kidneys ("turkey egg kidneys"), bladder, spleen, and larynx. Which one of the following choices is the most likely diagnosis? A. Haemophilus parasuis B. Hemagglutinating encephalomyelitis C. Classical swine fever D. Postweaning multisystemic wasting syndrome E. Anthrax - CORRECT ANSWER: C. Classical swine fever A pig farm reports a problem in weaning-age piglets. The affected piglets are depressed and anorexic with slight fevers. A few of the more severely-affected piglets have an exudative epidermitis affecting the face, limbs, and ventral abdomen. What should be recommended? A. High-dose antibiotics for sick piglets B. Cull affected animals C. Vaccinate herd with modified live vaccine D. Report to state vet E. Topical coumaphos/disinfectant combination dip - CORRECT ANSWER: A. High- dose antibiotics for sick piglets A pig farmer complains of strange behavior in his feeder pigs. Most are hyperexcitable yet are not squealing. A few are lethargic, wandering aimlessly and seem to be blind. Bloodwork shows a marked increase in sodium concentration. Upon inspection of the pen, it turns out that the waterer had been inadvertently turned off. Which one of the following treatments is indicated? A. Frequent small amounts of water B. Ad lib water C. Mannitol D. Furosemide E. Ad lib water and IV 1/2 strength Ringers solution - CORRECT ANSWER: A. Frequent small amounts of water A positive serology for Sarcocystis neurona antibodies in the serum of a horse indicates which one of the following? A. Favorable response to treatment for equine protozoal myeloencephalitis B. Definitive diagnosis of equine protozoal myeloencephalitis C. Poor response to treatment for equine protozoal myeloencephalitis D. Exposure to Sarcocystis neurona E. The horse has been in Africa - CORRECT ANSWER: D. Exposure to Sarcocystis neurona A pregnant mare was brought out to your barn for observation in anticipation of parturition. After several hours of restless behavior, several gallons of allantoic fluid rush out from the vulva. Which of the following would you expect to happen next for a normal parturition? A. The hind legs of the foal emerge from the vulva B. The red, velvety, chorioallantoic membrane emerges from the vulva C. The placenta is expelled from the vulva D. The thin, white, glistening amniotic membrane emerges from the vulva - CORRECT ANSWER: D. The thin, white, glistening amniotic membrane emerges from the vulva A pregnant mare was brought out to your barn for observation in anticipation of parturition. After several hours of restless behavior, several gallons of allantoic fluid rush out from the vulva. Which of the following would you expect to happen next for a normal parturition? A. The red, velvety, chorioallantoic membrane emerges from the vulva B. The thin, white, glistening amniotic membrane emerges from the vulva C. The placenta is expelled from the vulva D. The hind legs of the foal emerge from the vulva - CORRECT ANSWER: B. The thin, white, glistening amniotic membrane emerges from the vulva A rabbit is presented with inflamed and chapped membranes of the anus and genital region. The genital area is scalded and raw, with brownish crusts and purulent exudate. What two conditions top the differential diagnosis list? A. Tularemia, cystitis B. Treponematosis, hutch burn C. Myxomatosis, moist dermatitis D. Glomerulonephritis, coccidiosis E. Pasteurellosis, ulcerative pododermatitis - CORRECT ANSWER: B. Treponematosis, hutch burn A rescued Peking duck is presented for non-weight bearing lameness of the left pelvic limb and fever. Which one of the following choices is the most likely diagnosis? A. Septic arthritis B. Vitamin E deficiency C. Lead toxicosis D. Organophosphate toxicosis E. Rickettsial infection - CORRECT ANSWER: B. Vitamin E deficiency Acromegaly is associated with what 3 diseases in cats? A. Hypoadrenocorticism, Congestive heart failure, Hepatic lipidosis B. Hyperadrenocorticism, Pleural effusion, Lameness C. Diabetes mellitus, Cardiomyopathy, Renal disease D. Prognathism, Pulmonary edema, Pulmonary Hypertension E. Vertebral spondylosis, 3rd degree heart block, Glomerulonephritis - CORRECT ANSWER: C. Diabetes mellitus, Cardiomyopathy, Renal disease After the initial estrous cycle, a 6-month-old cat has firm enlargement of all mammary glands. Mammary gland hyperplasia is suspected. Which of the following is the most appropriate management? A. Bilateral radical mastectomy B. Estradiol cypionate therapy C. Furosemide therapy D. Megestrol acetate therapy E. Ovariohysterectomy - CORRECT ANSWER: E. Ovariohysterectomy An 8 year-old spayed female DSH cat is presented for right head tilt of 24 hours duration. The cat is up to date on vaccinations. Physical exam shows horizontal nystagmus with the fast phase to the left. The rest of the physical exam (including otoscopic exam) is unremarkable. What can the owner be told regarding the cat's treatment and prognosis? A. Excellent prognosis without medication B. Good prognosis with corticosteroids and anti-emetics C. Excellent prognosis with corticosteroid therapy D. Excellent prognosis with corticosteroids and antibiotics E. Fair prognosis with corticosteroids and antibiotics - CORRECT ANSWER: A. Excellent prognosis without medication -Idiopathic vestibular disease An 8-week old Abyssinian cat recently obtained from a cattery presents to you for an examination and the owner reports that the cat has had diarrhea. On fecal float, you find multiple structures like the one shown in the photo (see image). What should you treat the cat with? A. Sulfadimethoxine (Albon) B. Praziquantel (Droncit) C. Metronidazole (Flagyl) D. Pyrantel (Strongid) E. Selamectin (Revolution) F. Amoxicillin and clavulanate (Clavamox) - CORRECT ANSWER: A. Sulfadimethoxine (Albon) An 8-year old male tomcat presents for having a one week history of progressive lethargy, anorexia, occasional sneezing, conjunctivitis, and nasal discharge. On physical exam you see several ulcerations in the mouth. What is the most likely diagnosis? A. Bordetella bronchiseptica B. Foreign body C. Feline herpesvirus D. Feline calicivirus E. Chlamydophila felis - CORRECT ANSWER: D. Feline calicivirus An 11-year old female Pomeranian presents to you for coughing and exercise intolerance. On exam: Wt: 9.25 lbs, T: 101.2F (38.4 C), HR: 132 bpm, RR: Panting, mucous membranes are pink. She has mild tracheal sensitivity and a grade III-IV/VI left apical holosystolic murmur and grade II/VI right apical holosystolic murmur. Femoral pulses are strong and synchronous, with a regular rhythm. She has harsh lung sounds bilaterally. On abdominal palpation, you note hepatomegaly. You find bilateral luxating patellas. You take chest radiographs (see image) and decide to treat the dog based on these findings. Which medication plan is most appropriate? A. Clavamox and enrofloxacin B. Oxygen and atenolol C. Furosemide and enalapril D. Atropine and a temporary pacemaker E. Immiticide (Melarsomine) - CORRECT ANSWER: C. Furosemide and enalapril An 11-year old Peruvian Paso presents with a history of progressive weight loss. Serum chemistry shows elevation in sorbitol dehydrogenase, lactate, alkaline phosphatase, and a decrease in albumin. A vast amount of Crotalaria spp. is seen in the pasture. What type of toxin does Crotalaria spp. possess? A. Pyrrolizidine alkaloid B. Organophosphate C. Nitrate D. Cyanide - CORRECT ANSWER: A. Pyrrolizidine alkaloid An 11-year-old male, neutered Cavalier King Charles spaniel is presented for an episode of acute collapse with inability to support himself on the hind limbs and disorientation. Based on the radiographs and history, which one of the following tests would be next indicated? A. Magnetic resonance imaging B. Edrophonium challenge test C. Myelography D. Bronchoscopy E. Echocardiography - CORRECT ANSWER: E. Echocardiography Radiographs showed a severely enlarged cardiac silhouette An 11-year-old spayed female cat is presented with generalized weakness and is unable to lift her head. The owners have noticed her at the water bowl more often and she seems to be urinating more in the last month. Which blood chemistry test would be most likely to diagnose cause of her weakness? A. Sodium B. Potassium C. Chloride D. Calcium E. Bicarbonate - CORRECT ANSWER: B. Potassium An adopted adult cat from a shelter with an unknown vaccination history tests positive for feline leukemia virus (FeLV) infection by IFA. What should the owner be told? A. Possible transient FeLV infection B. Need an ELISA test in 12 weeks to confirm diagnosis C. Cat may be vaccinated for FeLV D. Need a Western blot test to confirm diagnosis E. Strong chance of lifelong FeLV infection - CORRECT ANSWER: E. Strong chance of lifelong FeLV infection An adult horse with clinical signs of voluminous gastric reflux, depression, colic, and fever is most likely suffering from which condition? A. Ulcerative duodenitis B. Proliferative enteropathy C. Proximal enteritis D. Right dorsal colitis E. Cantharidin toxicity - CORRECT ANSWER: C. Proximal enteritis An aged intact female potbellied pig presents for evaluation of abdominal distension. A large mass contiguous with the uterine wall is discovered with ultrasound exam of the abdomen. Which of the following choices is the most likely cause of this finding? A. Uterine leiomyoma B. Endometritis C. Adenocarcinoma of the cervix D. Lymphosarcoma E. Endometrial hyperplasia - CORRECT ANSWER: A. Uterine leiomyoma An immunochromatographic test kit for detection of fecal canine parvoviruses (CPV) antigen is being tested in a local cat shelter where as many as 10% of the cats there may have panleukopenia secondary to infection with the canine parvovirus. Here are simulated test results, compared to a gold standard test for CPV. CPV pos CPV neg Total Test kit positive 128 734 862 Test kit negative 63 1575 1638 Total 191 2309 2500 What is the sensitivity of this test kit? A. 734/862 Cats with taurine deficiency develop which of the following? A. Hypertrophic cardiomyopathy B. Dilated cardiomyopathy C. Myocarditis D. Restrictive cardiomyopathy - CORRECT ANSWER: B. Dilated cardiomyopathy Cats with taurine deficiency develop which of the following? A. Restrictive cardiomyopathy B. Hypertrophic cardiomyopathy C. Dilated cardiomyopathy D. Myocarditis - CORRECT ANSWER: C. Dilated cardiomyopathy Closure of the eyes is mediated by cranial nerve ____ and opening the eyes is mediated by CN ____. A. V, III B. III, VII C. V, VII D. VII, V E. VII, III - CORRECT ANSWER: E. VII, III Damage to the left recurrent laryngeal nerve is associated with "roaring" in horses. The left recurrent laryngeal nerve is a branch off of which cranial nerve? A. Glossopharyngeal (CN 9) B. Facial (CN 7) C. Trigeminal (CN 5) D. Hypoglossal (CN 12) E. Vagus (CN 10) - CORRECT ANSWER: E. Vagus (CN 10) During a routine immunization visit for a 2 year-old neutered male Newfoundland dog, a systolic ejection-type (crescendo-decrescendo) murmur is detected. It is audible loudest on the left side of the chest between the 2nd and 5th intercostal (IC) space and at the thoracic inlet lateral to the trachea. Which condition is highest on a differential diagnosis list? A. Pulmonic stenosis B. Mitral dysplasia C. Tricuspid dysplasia D. Aortic stenosis E. Patent ductus arteriosus (PDA) - CORRECT ANSWER: D. Aortic stenosis During the necropsy of an 8-year-old mixed breed dog from the Southern United States, reactive granulomas in the esophagus containing bright red worms, 40 mm to 70 mm long are found. Which choice is the most likely diagnosis? A. Ollulanus tricuspis B. Haemonchus placei C. Spirocerca lupi D. Gastrophilus spp. E. Physaloptera spp. - CORRECT ANSWER: C. Spirocerca lupi During your examination of an 18-year old horse, you observe what is shown in the photograph. Based on the location and appearance of this lesion, what is the most likely diagnosis? (Photo showed darkly pigmented lesions in the perineal region and on the ventrum of the tail) A. Sarcoid B. Squamous cell carcinoma C. Melanoma D. Habronema E. Cuterebra - CORRECT ANSWER: C. Melanoma Failure to control mild endemic respiratory disease in swine caused by Mycoplasma hyopneumoniae predisposes pigs to complications. Exudative bronchopneumonia and polyarthritis are most commonly seen in herds infected with both Mycoplasma hyopneumoniae AND which other organism? A. Bordetella bronchiseptica B. Fusobacterium necrophorum C. Hemophilus parasuis D. Pasteurella multocida E. Swine influenza virus - CORRECT ANSWER: D. Pasteurella multocida Feline leukemia (FeLV) status has been shown to affect many aspects of disease progression and treatment of lymphoma. FeLV seropositive cats with lymphoma are similar to FeLV seronegative cats with lymphoma in which one of the following aspects? A. Frequency of spinal lymphoma B. Initial treatment response C. Survival time D. Prevalence of gastrointestinal lymphoma E. Prevalence of mediastinal lymphoma - CORRECT ANSWER: B. Initial treatment response Foal heat diarrhea is typically associated with which choice? A. Hemorrhagic enteritis B. Neutropenia and fever C. Decreased suckling D. Alterations in diet E. Warm and humid weather - CORRECT ANSWER: D. Alterations in diet For the last 10 years, your state has had a mandatory vaccination program against "pedunculated giblet disease" in fur-bearing turtles and the prevalence of this terrible disease has decreased markedly. How does this decrease in prevalence affect the predictive value positive (PVP) of the best serologic test for pedunculated giblet disease? A. PVP depends on the number tested, not prevalence B. PVP is affected by specificity, not prevalence C. PVP increases as prevalence decreases D. PVP decreases as prevalence decreases E. PVP stays the same as prevalence decreases - CORRECT ANSWER: D. PVP decreases as prevalence decreases Fractures of the proximal sesamoid bones in horses are often associated with damage to which structure? A. Suspensory ligament B. Superior check ligament C. Deep digital flexor tendon D. Superficial digital flexor tendon E. Impar ligament - CORRECT ANSWER: A. Suspensory ligament Fumonisin intoxication is primarily associated with which clinical signs? A. Leukoencephalomalacia, hypertension B. Salivation, retching C. Estrogenism, vulvovaginitis D. Vomiting, immunosuppression E. Terminal necrosis of extremities, gangrene - CORRECT ANSWER: A. Leukoencephalomalacia, hypertension If a mass appears in the lungs on a right lateral radiograph but not on a left lateral radiograph, where is the mass located? A. Body wall B. Right lung C. Mediastinum D. Left lung - CORRECT ANSWER: D. Left lung If a mass appears in the lungs on a right lateral radiograph but not on a left lateral radiograph, where is the mass located? A. Left lung B. Mediastinum C. Body wall D. Right lung - CORRECT ANSWER: A. Left lung Imidocarb diproprionate is the most effective therapeutic for which one of the following organisms? A. Bartonella spp. B. Babesia canis C. Rickettsia rickettsii D. Borrelia burgdorferi E. Trypanosoma congolense - CORRECT ANSWER: B. Babesia canis In canine patients with primary hyperparathyroidism, a chemistry panel would show: swelling of the head and neck. A few have paralyzed legs and wings, twisted necks, are circling, and have tremors or clonic spasms. Birds are not laying well and some eggs are misshapen with watery albumen. Many of the sickest birds have died. Necropsy of a dead chicken shows petechial hemorrhages on the mucosal surface of the proventriculus and gizzard. What should be done next to address this problem? A. Increase poultry house temperatures to reduce morbidity B. Disinfect housing with phenolic compounds, barrier precautions for staff C. Treat all affected chickens with amprolium D. Cull affected birds and vaccinate the remainder E. Report outbreak to regulatory authorities - CORRECT ANSWER: E. Report outbreak to regulatory authorities Newcastle's disease Many turkeys on a poultry farm develop whitish "wart-like" nodules and scabs on the comb, wattles, feet, and vent. Which management intervention would help prevent spread of the disease? A. Immediate removal of fecal waste B. Raise the room temperature 5 degrees C. Mosquito control D. Add antibiotics to the drinking water E. Thoroughly disinfect pens and equipment F. Tick control - CORRECT ANSWER: C. Mosquito control Many turkeys on a poultry farm develop whitish "wart-like" nodules and scabs on the comb, wattles, feet, and vent. Which management intervention would help prevent spread of the disease? A. Mosquito control B. Add antibiotics to the drinking water C. Tick control D. Thoroughly disinfect pens and equipment E. Raise the room temperature 5 degrees F. Immediate removal of fecal waste - CORRECT ANSWER: A. Mosquito control Disease described is a form of avian (fowl) pox, which is spread by mosquitoes Once infected, for what period of time is canine parvovirus usually shed? A. 2-3 months B. 5-6 months C. 3-4 weeks D. 7-10 days - CORRECT ANSWER: D. 7-10 days One description of a typical heart sound is "lub-dub." What makes the first heart sound (S1) (i.e., the "lub")? A. Ventricular filling B. Aortic and pulmonic valves C. Mitral and semilunar valves D. Atrial contraction E. Atrioventricular valves - CORRECT ANSWER: E. Atrioventricular valves Oral administration of which of these drugs has been implicated as a cause of esophageal strictures in cats? A. Potassium bromide B. Doxycycline C. Azithromycin D. Diazepam - CORRECT ANSWER: B. Doxycycline Over the past 2 weeks, several pigs in a herd have been febrile and depressed. Many were constipated, then had diarrhea. A few were incoordinated and one had seizures. A few have died. Necropsy revealed petechial hemorrhages on the kidneys and larynx, and a hemorrhagic urinary bladder. Of the following choices, which one is most consistent with a presumptive diagnosis? A. Glasser's disease B. Erysipelas C. Swine dysentery D. Classical swine fever E. Streptococcus suis infection - CORRECT ANSWER: D. Classical swine fever Oxalate is the toxic agent found in which one of the following plants? A. Hypericum perforatum (klamathweed) B. Delphinium spp. (larkspur) C. Sarcobatus vermiculatus (greasewood) D. Hordeum spp. (foxtail) E. Solanum spp. (nightshade) - CORRECT ANSWER: C. Sarcobatus vermiculatus (greasewood) Pemphigus foliaceus, pemphigus vulgaris, and bullous pemphigoid are examples of which type of immune-mediated disease? A. Type II B. Delayed hypersensitivity C. Type IV D. Type III E. Type I - CORRECT ANSWER: A. Type II Prior to the 1950s, heat pasteurization of milk was done principally to prevent transmission of one organism to people. Today, that organism is re-emerging in bi- national communities with ties to Mexico who consume unpasteurized milk products. What is the organism? A. Mycobacterium bovis B. Enterotoxigenic Escherichia coli O157:H7 C. Mycobacterium avium subspecies paratuberculosis D. Mycobacterium tuberculosis E. Mycobacterium avium - CORRECT ANSWER: A. Mycobacterium bovis Pyrrolizidine alkaloid toxicity is caused by chronic ingestion of which one of the following plants? A. Astragalus spp. (locoweed) B. Lupinus spp. (lupine) C. Nerium spp. (oleander) D. Senecio spp. (ragwort) E. Persea spp. (avocado) - CORRECT ANSWER: D. Senecio spp. (ragwort) Recently, several chickens in a free-range flock have died. Many others have not been eating as well, look unthrifty, and are less active than they used to be. Necropsy of one of the chickens that died reveals marked thickening and inflammation of the crop and esophageal mucosae. The following parasite eggs are identified in a sample from the litter in the henhouse. What is the most likely parasite affecting this flock? A. Heterakis gallinarum B. Ascaridia galli C. Syngamus trachea D. Davainea proglottina E. Capillaria contorta - CORRECT ANSWER: E. Capillaria contorta Retained placenta and metritis can predispose to which one of the following secondary conditions in horses? A. Laminitis B. Postpartum dysgalactia syndrome C. Contagious equine metritis D. Cystic endometriosis E. Colic - CORRECT ANSWER: A. Laminitis Roughly 10% of the dry sows in a herd of pigs have poor appetite, fever, and are coughing. There have been several late-term abortions and an increase in returns post- service. Some of the farrowing sows have poor appetite and thirst. Several have mastitis and/or agalactia and are farrowing 23 days early. Some piglets born alive have diarrhea, and some in the litter were born dead and mummified. There's been more respiratory disease in the piglets recently and the farmer has heard a funny thumping noise when the most severely affected piglets breath. The farmer noticed that several of the sows have transiently had blue ears. Based on the presumptive diagnosis, if this disease becomes endemic in the herd, which of the following is a likely sequela? A. Necrotic ear syndrome in piglets B. Enzootic pneumonia in grower/finisher units C. Chronic diarrhea in breeding sows D. Neurologic syndromes in breeding boars E. Diamond- - CORRECT ANSWER: B. Enzootic pneumonia in grower/finisher units Rubiosis iridis as seen in this cat is a sign of what process in the eye? A. Lens luxation B. Anterior uveitis C. Iris atrophy Several piglets in a group weaned 10 days ago in the nursery facility of a large commercial swine operation were found dead. On evaluation, some weaners have swelling around the eyes and forehead. Some are in lateral recumbency and dyspneic. Necropsy of the dead piglets reveals subcutaneous and submucosal edema. The most likely causative organism is... A. Brachyspira hyodysenteriae B. Escherichia coli C. Lawsonia intracellularis D. Clostridium septicum E. Streptococcus suis - CORRECT ANSWER: B. Escherichia coli Several sheep are sick at a petting-zoo that has cows, horses, pigs, bison and white- tailed deer. One of the deer is also affected. Affected sheep are listless and off-feed, with serous or mucopurulent nasal discharge and rectal temperatures ranging from 105- 107.5°F (40.5-42°C). Physical exam shows swollen muzzles with edema and congestion of the lips, nose and face with small hemorrhages and ulcers on the mucous membranes. The ulcers appear where the teeth contact the swollen lips and tongue. Two affected sheep are lame. What is the diagnosis? A. Bluetongue B. Peste des petit* ruminants (PPR) C. Caprine arthritis encephalitis D. Contagious ecthyma (soremouth) E. Foot-and-mouth disease (FMD) - CORRECT ANSWER: A. Bluetongue Several younger sheep in a large herd have died suddenly with a diagnosis of infectious necrotic hepatitis secondary to fluke infestation. Which choice is the most practical and effective control measure you can take next to prevent future cases? A. Vaccinate all animals with Clostridium novyi toxoid B. High-dose penicillin for clinical cases C. Clostridium hemolyticum bacterin for animals under 3 years D. Clorsulon antihelmintic for whole herd E. Treat ponds with copper sulfate molluscicide against lymnaeid snails - CORRECT ANSWER: A. Vaccinate all animals with Clostridium novyi toxoid Severe combined immunodeficiency is a lethal autosomal recessive trait in Arabian foals. Heterozygotes are clinically normal. If the heterozygote carrier rate for the genetic mutation is 8%, what is the expected frequency of Arabian foals that are hom*ozygous for the mutated allele? A. 25% B. 0.16% C. 0.64% D. 4% E. 2% F. 0.064% - CORRECT ANSWER: C. 0.16% Both parents must be carriers (heterozygous) --> 0.08 x 0.08 = 0.0064 or 0.64% Offspring then has a 1 in 4 chance of being hom*ozygous for disease --> 0.0064 x 0.25 = 0.0016 or 0.16% Severe combined immunodeficiency is a lethal autosomal recessive trait in Arabian foals. Heterozygotes are clinically normal. If the heterozygote carrier rate for the genetic mutation is 8%, what is the expected frequency of Arabian foals that are hom*ozygous for the mutated allele? A. 25% B. 4% C. 0.064% D. 0.16% E. 2% F. 0.64% - CORRECT ANSWER: D. 0.16% Chance of both parents being carriers = 8% x 8% (0.8 x 0.8 = 0.0064 or 0.64%) 1 in 4 chance of inheriting two mutant alleles, therefore overall expected frequency of diseased foals = 0.0064 x 0.25 = 0.0016 or 0.16% Suzie-Q, a 6-month old female spayed domestic short hair was recently adopted from the humane society. She has had watery diarrhea since adoption. Her fecal float and Giardia ELISA tests were negative. She was treated with metronidazole with no clinical improvement. You soak a cotton tip swab with saline and swab the rectum. You see elongated motile oval shaped protozoan organisms that do not look like Giardia lamblia. What organism might this be and what is the appropriate therapy? A. Taenia taeniaformis, Praziquantel B. Enterobius vermicularis, Fenbendazole C. Giardia intestinalis, Fenbendazole D. Cryptosporidium, Clindamycin E. Tritrichom*onas foetus, Ronidazole F. Paragonimus kellicotti, Praziquantel - CORRECT ANSWER: E. Tritrichom*onas foetus, Ronidazole The 12-year-old spayed female cat shown below is presented with a history of weakness for the past 2 days. She has neck ventroflexion and a stiff, stilted gait. Which one of the following is in the top of the differential list? A. Hypokalemic myopathy B. Hyperkalemic periodic paralysis C. Cervical vertebral malformation D. Caudal occipital malformation E. Bilateral otitis media/interna - CORRECT ANSWER: A. Hypokalemic myopathy The cat in the image below presents for lethargy, depression, and weakness. The cat can't seem to lift his head (as seen in this image). You recommend running a chemistry panel on the cat. What potential finding explains can explain the findings? A. Low potassium B. Low phosphorus C. High urea nitrogen D. High glucose E. Low calcium - CORRECT ANSWER: A. Low potassium The dog in the photograph was vaccinated earlier in the day. What type of allergic reaction is occurring? A. Type IV B. Type I C. Type III D. Type II - CORRECT ANSWER: B. Type I The goose shown in the image below was found weak on the shore of a local pond where oil had been dumped. The goose was covered in oil, dehydrated and weak but responsive. Which of the following is an important acute clinical effect of oil on affected birds? A. Nephrotoxicity B. Hepatotoxicity C. Disruption of function of the plumage D. Contact dermatitis E. Lead toxicity - CORRECT ANSWER: C. Disruption of function of the plumage The image shows an infarct in the liver discovered on post mortem exam of a mature beef cow which died one hour before in a western mountain pasture, after being observed to appear normal one day earlier. There is also dark red urine in the bladder. The pasture contains native plants, some pine trees, and a marshy area with water plants. The cows are unvaccinated and were never wormed. Given this history and the lesion found, the most likely cause of death is _________. A. Death camas toxicity B. Pine needle poisoning C. Blue-green algae toxicity D. Bacillary hemoglobinuria E. Viral hepatitis - CORRECT ANSWER: D. Bacillary hemoglobinuria The image shows an infarct in the liver discovered on post mortem exam of a mature beef cow which died one hour before in a western mountain pasture, after being observed to appear normal one day earlier. There is also dark red urine in the bladder. The pasture contains native plants, some pine trees, and a marshy area with water plants. The cows are unvaccinated and were never wormed. Given this history and the lesion found, the most likely cause of death is _________. A. Viral hepatitis B. Blue-green algae toxicity C. Pine needle poisoning D. Death camas toxicity E. Bacillary hemoglobinuria - CORRECT ANSWER: E. Bacillary hemoglobinuria The major active ingredient in most IV euthanasia solutions is: E. Biopsy - CORRECT ANSWER: A. Fungal culture of hair and skin scales This 3-year-old horse is presented for evaluation of this dry, horny, wart-like mass on the distal forelimb. This horse also has smaller wart-like lesions on its muzzle. The other young horses in the herd have similar lesions. Which one of the following is the most likely diagnosis? A. Trombiculosis B. Melanomatosis C. Dermatophilosis D. Pediculosis E. Papillomatosis - CORRECT ANSWER: E. Papillomatosis This is a corneal abscess secondary to infectious keratoconjunctivitis (pinkeye). What is the most commonly recognized organism that causes pinkeye in cows? A. Colesiota conjunctivae B. Chlamydophila pecorum C. Mycoplasma spp. D. Moraxella bovis E. Neisseria spp. - CORRECT ANSWER: D. Moraxella bovis This plant, yellow star thistle, is a substantial source of forage for a herd of goats. What would be the most likely problem seen in the goats? A. Hemorrhagic gastroenteritis B. Hepatic fibrosis and megalocytosis C. Diffuse fibrosing alveolitis D. Nigropallidal encephalomalacia E. No illness - CORRECT ANSWER: E. No illness Tommy, a 4-year old male neutered domestic short hair, presents to you for frequent urination. Urinalysis reveals 4+ struvite crystals. An abdominal radiograph shows a 1 cm round calculus in the bladder. Tommy does not have a urethral blockage, and urine culture is negative. The owner says surgery to remove this stone is not an option due to finances. Which of the following would be the most important treatment for Tommy? A. Acidifying diet B. Clavamox C. Alkalinizing diet D. Potassium citrate E. Metacam - CORRECT ANSWER: A. Acidifying diet Two bucket-fed veal calves are depressed and stunted and have sticky feces coating their hindquarters. Fluid-splashing sounds are audible during auscultation over the left flank when a calf is drinking. Passage of a stomach tube obtains a rancid-smelling liquid with a pH of 5.2 [N=5.9-6.2] Which one of the following choices is the most appropriate treatment? A. Remove fermented rumen contents, flush with saline B. Wean affected calves; Shift to bottle-feeding for well calves C. Vitamin E/Selenium injections, plus dietary supplements D. Cull affected calves E. Inoculate rumen fluid from a healthy cow into calf by stomach tube - CORRECT ANSWER: A. Remove fermented rumen contents, flush with saline Two calves aged eight weeks are presented that are both down and extremely weak. They are depressed and lying in pools of foul-smelling brown diarrhea with a small amount of blood. One other calf died suddenly the night before with no signs at all. They are found to be dehydrated, with rectal temperatures of 105.2 and 105.6 F (40.7 - 40.9 C)..[N=101.5-103.5F, N=37.8-39.7 C], respectively. Based on the condition at the top of the differential diagnosis list, what is the treatment plan? A. Immunize well and sick calves and adult cattle with MLV vaccine, antibiotics for sick calves B. NSAIDS, antibiotics in feed and water C. IV fluids, NSAIDS, frequent milk feeding, antibiotics if septic D. Cull sick calves, disinfect feeding areas, prophylactic antibiotics and NSAIDs for well animals E. Isolate sick calves, immunize, antibiotics, cull those that do not respond to treatment - CORRECT ANSWER: C. IV fluids, NSAIDS, frequent milk feeding, antibiotics if septic Under what conditions is a very sensitive test used? A. Rare disease, Early diagnosis improves prognosis B. Lethal disease, Highly prevalent disease C. Treatment does not affect prognosis, Non-infectious diseases D. Common disease, infectious diseases E. Zoonoses, untreatable diseases - CORRECT ANSWER: A. Rare disease, Early diagnosis improves prognosis Highly seNsitive test = few false Negatives!! What acid-base abnormality would be most likely in a dairy cow with a displaced abomasum? A. Metabolic alkalosis B. Metabolic acidosis C. Depends on severity of displacement D. Respiratory alkalosis E. Respiratory acidosis - CORRECT ANSWER: A. Metabolic alkalosis What are ongoing losses in fluid therapy? A. Urine output B. Fluids given minus extracellular volume estimate C. Fluid lost during panting D. Vomit and diarrhea E. Percent dehydration multiplied by body weight - CORRECT ANSWER: D. Vomit and diarrhea What distinguishes lymphoma from leukemia? A. Lymphoma is benign B. Leukemia is derived only from myeloid cells C. Leukemia is the precursor to lymphoma D. Lymphoma occurs only in dogs E. Lymphoma originates in solid tissues - CORRECT ANSWER: E. Lymphoma originates in solid tissues What is a common side effect of xylazine administration in cats? A. Anuria B. Vomiting C. Polycythemia D. Miosis E. Seizures - CORRECT ANSWER: B. Vomiting What is a common side effect of xylazine administration in cats? A. Vomiting B. Seizures C. Anuria D. Polycythemia E. Miosis - CORRECT ANSWER: A. Vomiting What is a gilt? A. Castrated male sheep B. Female pig that has not had a litter yet C. A male pig, castrated before puberty D. Ewe that has lambed at least once E. Female pig that has had more than one litter - CORRECT ANSWER: B. Female pig that has not had a litter yet What is the average length of estrus in the cat? A. 2-3 days B. 4-5 days C. 9-10 days D. 6-7 days E. 21 days - CORRECT ANSWER: D. 6-7 days What is the earliest stage post-breeding that an experienced practitioner can reliably feel fremitus rectally in the uterine artery of the gravid uterine horn in a pregnant cow? A. 60-75 days B. 45-60 days C. 70-80 days D. 120-150 days E. 90-120 days - CORRECT ANSWER: E. 90-120 days What is the main mode of transmission for feline leukemia virus? A. Fomite transmission What is the primary factor associated with pregnancy toxemia in ewes and goats? A. Young age at parturition B. Clostridium perfringens C. Polioencephalomalacia D. Inadequate nutrition E. High sulfur, low thiamine diet - CORRECT ANSWER: D. Inadequate nutrition What is the primary mode of transmission of West Nile Virus (WNV) between birds? A. Via feces B. Via Cnemidocoptes mites C. Via placenta D. Via saliva E. Via mosquitoes - CORRECT ANSWER: E. Via mosquitoes What is the primary mode of transmission of West Nile Virus (WNV) between birds? A. Via saliva B. Via mosquitoes C. Via Cnemidocoptes mites D. Via placenta E. Via feces - CORRECT ANSWER: B. Via mosquitoes What is the proper name for the swine kidney worm? A. Capillaria plica B. Stephanurus dentatus C. Dioctophyma renale D. Gnathostoma spinigerum E. Trichinella spiralis - CORRECT ANSWER: B. Stephanurus dentatus What kind of organism causes equine granulocytic ehrlichiosis? A. Spirochete B. Anaplasma C. Ehrlichia D. Protozoa E. Chlamydia - CORRECT ANSWER: B. Anaplasma What percentage of dairy cows have a retained placenta after calving? A. 1-4% B. 5-15% C. 25% D. 20-30% E. 30-40% - CORRECT ANSWER: B. 5-15% What profile would you expect in a dog with hypervitaminosis D? A. High Ca, Low P B. High Ca, High P C. Low Ca, High P D. Low Ca, Low P - CORRECT ANSWER: A. High Ca, High P What profile would you expect in a dog with hypervitaminosis D? A. High Ca, Low P B. High Ca, High P C. Low Ca, High P D. Low Ca, Low P - CORRECT ANSWER: B. High Ca, High P What species is the principle carrier and vector of Malignant Catarrhal Fever (MCF) virus in North America? A. Bison B. Horse C. Donkey D. Sheep E. Cattle - CORRECT ANSWER: D. Sheep What species is the principle carrier and vector of Malignant Catarrhal Fever (MCF) virus in North America? A. Bison B. Sheep C. Donkey D. Cattle E. Horse - CORRECT ANSWER: B. Sheep What tumor type is a cat predisposed to developing at vaccination sites? A. Fibrosarcoma B. Squamous cell carcinoma C. Melanoma D. Lymphoma - CORRECT ANSWER: A. Fibrosarcoma What tumor type is a cat predisposed to developing at vaccination sites? A. Squamous cell carcinoma B. Fibrosarcoma C. Lymphoma D. Melanoma - CORRECT ANSWER: B. Fibrosarcoma When a foal is being treated with erythromycin (for Rhodococcus equi, for example), the mare is at risk of developing enterocolitis due to which one of the following organisms? A. Escherechia coli B. Clostridium novyi C. Lawsonia intracellularis D. Rhodococcus equi E. Clostridium difficile - CORRECT ANSWER: E. Clostridium difficile When a sow is giving birth, what is the shortest time interval between passage of piglets that indicates dystocia? A. 4 hours B. 1/2 hour C. 2 hours D. 1 hour E. 15 minutes - CORRECT ANSWER: D. 1 hour When performing a subtotal colectomy on a feline patient, what blood vessel limits the amount of colon that you are able to remove? A. Caudal mesenteric artery B. Pudendal artery C. Ileocolic artery D. Left colic artery - CORRECT ANSWER: C. Ileocolic artery When performing a subtotal colectomy on a feline patient, what blood vessel limits the amount of colon that you are able to remove? A. Pudendal artery B. Caudal mesenteric artery C. Ileocolic artery D. Left colic artery - CORRECT ANSWER: C. Ileocolic artery Where is the predominant abnormality located in these radiographs from an 11 year old English sheepdog? A. Spleen B. Pancreas C. Stomach D. Right kidney E. Liver - CORRECT ANSWER: C. Stomach Radiographs showed a markedly enlarged stomach that was filled with fluid, gas, and a mineral opacity material with sharp edges. Which animal is considered to be as susceptible to acetaminophen toxicity as cats? A. Dogs B. Rabbits C. Hamsters D. Ferrets E. Rats - CORRECT ANSWER: D. Ferrets Which choice describes inflammation of an acquired bursa over the dorsal carpus in the horse? A. Villonodular synovitis B. Ulceroproliferative stomatitis C. Atheroma D. Hygroma E. Suspensory desmitis - CORRECT ANSWER: D. Hygroma D. Administration of a fleet enema to a cat E. Intoxication with cholecalciferol based rodenticide - CORRECT ANSWER: E. Intoxication with cholecalciferol based rodenticide Which of the following drugs is known for stimulating appetite in small animals? A. Tetracycline B. Acyclovir C. Omeprazole D. Cyproheptadine - CORRECT ANSWER: D. Cyproheptadine Which of the following intravenous solutions are hypotonic? A. 0.45% Saline, 5% dextrose B. Lactated ringers solution, Normosol-R® C. Hetastarch, Vetstarch D. Plasmalyte 148, Normosol-M® E. 5% Dextrose in 0.9% saline - CORRECT ANSWER: A. 0.45% Saline, 5% dextrose Which of the following is a causative agent of infectious bovine keratoconjunctivitis as shown in this image? A. Histophilus somni B. Moraxella bovis C. E. coli D. Thelazia - CORRECT ANSWER: B. Moraxella bovis Which of the following is FALSE about Feline Immunodeficiency Virus? A. Prevalence is higher in free-roaming stray cats B. Affects all ages but is most prevalent in cats 6 years old cats and younger C. More prevalent in male cats D. Wild felids are susceptible - CORRECT ANSWER: B. Affects all ages but is most prevalent in cats 6 years old and younger Which of the following is incorrect concerning feline heartworm disease? A. Cats typically have much lower adult worm burdens than dogs B. A negative microfilaria test does not rule out heartworm disease C. Migration of larvae to ectopic regions (outside of the heart and pulmonary arteries) is less common than in dogs D. Antibody tests can detect the exposure of the host to both male and female worms - CORRECT ANSWER: C. Migration of larvae to ectopic regions (outside of the heart and pulmonary arteries) is less common than in dogs Which of the following is not a component of therapy for chronic renal failure? A. H2 receptor antagonists B. Glucocorticoids C. Oral phosphate binding agents D. Restricted dietary protein E. Calcium channel blocking agents - CORRECT ANSWER: B. Glucocorticoids Which of the following is not a treatment for feline hyperthyroidism? A. Methimazole B. Thyroxine C. Radioactive iodine therapy D. Thyroidectomy - CORRECT ANSWER: B. Thyroxine Which of the following is the causative agent of fowl cholera? A. Chlamydophila psittaci B. Mycoplasma gallisepticum C. Pasteurella multocida D. Reticuloendotheliosis virus - CORRECT ANSWER: C. Pasteurella multocida Which of the following is the causative agent of fowl cholera? A. Pasteurella multocida B. Chlamydophila psittaci C. Reticuloendotheliosis virus D. Mycoplasma gallisepticum - CORRECT ANSWER: A. Pasteurella multocida Which of the following is the correct location for placement of a proximal paravertebral block used to perform a standing laparotomy in a cow? A. In the spaces between L1, L2, L3, and L4 B. Above and below the transverse processes of L1, L2 and L4 C. Midway between spine and the ends of the transverse processes of L1, L2, L3 D. Caudal to the transverse processes of T13, L1, and L2 E. At the ends of the transverse processes of L1, L2, and L4 - CORRECT ANSWER: D. Caudal to the transverse processes of T13, L1, and L2 Which of the following is the most likely cause of dilated cardiomyopathy in an adult cat? A. Hyperthyroidism B. Neoplasia C. Pulmonary fibrosis D. Taurine deficiency E. Toxoplasmosis - CORRECT ANSWER: D. Taurine deficiency Which of the following is the most likely cause of generalized polymyopathy secondary to chronic renal failure in cats? A. Hypochloridemia B. Hypokalemia C. Hypomagnesemia D. Hyponatremia E. Hypophosphatemia - CORRECT ANSWER: B. Hypokalemia Which of the following nerves are targeted with a paravertebral block used to perform a standing laparotomy in a cow? A. L1-3, and S1-5 B. T13, L1, and L2 C. L2, L3, S1, and S2 D. L1, L2, L3 E. L1, L2, and L4 - CORRECT ANSWER: B. T13, L1, and L2 Which of the following pairs of tissues can both cause increased serum alanine aminotransferase (ALT) levels in dogs and cats? A. Pancreas and intestine B. Kidneys and pancreas C. Liver and muscle D. Spleen and adrenals E. Heart and kidneys - CORRECT ANSWER: C. Liver and muscle Which of the following zoonotic pathogens is most likely to be acquired via raw goat milk? A. Bacillus anthracis B. Brucella abortus C. Brucella melitensis D. Clostridium difficile E. Brucella suis - CORRECT ANSWER: C. Brucella melitensis Which of these are used in the minor cross match for blood products when looking for a compatible blood donor for a dog? A. Donor red blood cells, donor plasma B. Recipient red blood cells, donor plasma C. Recipient red blood cells, recipient plasma D. Donor red blood cells, recipient plasma - CORRECT ANSWER: B. Recipient red blood cells, donor plasma Which of these are used in the minor cross match for blood products when looking for a compatible blood donor for a dog? A. Donor red blood cells, recipient plasma B. Donor red blood cells, donor plasma C. Recipient red blood cells, donor plasma D. Recipient red blood cells, recipient plasma - CORRECT ANSWER: C. Recipient red blood cells, donor plasma Which of these drugs has the greatest potential for causing acute renal failure in the horse? A. Dexamethasone B. Oxytetracycline C. Neomycin D. Xylazine E. Diphenhydramine - CORRECT ANSWER: C. Neomycin B. Brome hay C. Timothy hay D. Alfalfa hay - CORRECT ANSWER: C. Timothy hay Which one of the following choices includes the cardinal sign of trigeminal neuritis? A. Masseter muscle pain associated with chewing B. Circling and head tilt toward side of lesion, no other signs C. Dysphagia, dysphonia and stridor D. Inability to close the mouth E. Paralyzed eyelid, ear or lip on one or both sides of the face - CORRECT ANSWER: D. Inability to close the mouth Which one of the following choices is a cardinal and early sign of toxicoinfectious botulism in a foal? A. When startled, affected foals fall and exhibit opisthotonos B. Milk dribbles from mouth and nose; foal cannot retract tongue C. Sawhorse stance, trismus, bilaterally prolapsed nictitans (3rd eyelid) D. Frequent chewing motions, hypersalivation, vocalizing E. Hemorrhagic enterocolitis with headpressing - CORRECT ANSWER: B. Milk dribbles from mouth and nose; foal cannot retract tongue Which one of the following choices is a competitive antagonist for aldosterone that is sometimes used in treatment of congestive heart failure? A. Hydrochlorothiazide B. Furosemide C. Mannitol D. Acetazolamide E. Spironolactone - CORRECT ANSWER: E. Spironolactone Which one of the following choices is a major clinical complication of equine pituitary pars intermedia dysfunction? A. Dyshormonogenetic goiter B. Circling, head-pressing C. Diabetes insipidus D. Colic E. Laminitis - CORRECT ANSWER: E. Laminitis Which one of the following choices is the best initial treatment in the patient with a moderate elevation of calcium caused by idiopathic feline hypercalcemia? A. Injectable fluids B. Prednisolone C. Furosemide D. Bisphosphonates E. Diet change - CORRECT ANSWER: E. Diet change Which one of the following choices is the best way to evaluate the body condition of an adult llama? A. Palpate over the lumbar vertebrae, withers B. Evaluate the space between the olecranon and the chest (armpit) C. Digital exam from the supraorbital fossa to masseter D. Rectal exam to evaluate pelvic fat E. Weight X height at shoulderblades X 0.28 - CORRECT ANSWER: A. Palpate over the lumbar vertebrae, withers Which one of the following choices is the mechanism of action of omeprazole? A. Cyclooxygenase blocker B. Beta-adrenergic receptor agonist C. Synthetic prostaglandin E1 analog D. H2-receptor antagonist E. Proton pump inhibitor - CORRECT ANSWER: E. Proton pump inhibitor Which one of the following choices is the most appropriate next step after a 3-year-old stray cat tests positive for Feline Immunodeficiency Virus (FIV) on a routine FIV ELISA screening test? A. Recheck serum by FIV complement fixation test B. Do a Rivalta test to rule out feline coronavirus-related interference C. Treat with glucocorticoids D. Perform for a Western blot FIV test E. Euthanize - CORRECT ANSWER: D. Perform for a Western blot FIV test Which one of the following choices is the most common etiology of canine cholangiohepatitis? A. Neoplastic biliary obstruction B. Idiopathic C. Ascending infection D. Liver fluke migration E. Toxicity - CORRECT ANSWER: C. Ascending infection Which one of the following conditions occurs in the Fall in temperate climates, among cattle moved from poor pasture to lush pasture? A. Rumenal hyperkeratosis B. Acute bovine pulmonary emphysema and edema C. Vena caval thrombosis and metastatic pneumonia D. Verminous bronchitis (Husk) E. Vagal indigestion - CORRECT ANSWER: B. Acute bovine pulmonary emphysema and edema Which one of the following diseases have been linked with the variant form of Creutzfeldt-Jakob disease (vCJD) in humans? A. Bovine spongiform encephalopathy B. Q fever C. Sporadic bovine encephalomyelitis D. Equine West Nile virus encephalitis E. Eastern equine encephalomyelitis (EEE) - CORRECT ANSWER: A. Bovine spongiform encephalopathy Which one of the following diseases is characterized by schistocytes on a complete blood count? A. Congenital hepatic arteriovenous fistula B. Disseminated intravascular coagulation C. Cyanocobalamin deficiency D. Methemoglobinemia E. Acetaminophen toxicity - CORRECT ANSWER: B. Disseminated intravascular coagulation Which one of the following hormones can induce neoplastic transformation of hypertrophied mammary tissue in cats? A. Oxytocin B. Progesterone C. Estrogen D. Testosterone E. Prolactin - CORRECT ANSWER: B. Progesterone Which one of the following is the most common cause of dystocia in cattle? A. Uterine inertia B. Fetal monsters C. Maternal birth canal abnormalities D. Abnormal fetal presentation, position, and/or posture E. Fetopelvic disproportion - CORRECT ANSWER: E. Fetopelvic disproportion Which one of the following organisms transmits the causative agent of blackhead disease in turkeys? A. Heterakis gallinarum B. Ascardia galli C. Capillaria spp D. Eimeria spp E. Raillietina spp - CORRECT ANSWER: A. Heterakis gallinarum Which one of the following vaccines is considered a core vaccine for South American camelids? A. Rabies B. Tetanus C. Listeria D. Equine herpes virus I E. Bovine viral diarrhea virus - CORRECT ANSWER: B. Tetanus A. Administer fenbendazole twice annually, once in the early summer and again in late summer B. Administer ivermectin twice annually, once in the early summer and again in the fall C. Administer ivermectin twice annually, once in the early spring and again in the winter D. Administer fenbendazole twice annually, once in the early spring and - CORRECT ANSWER: B. Administer ivermectin twice annually, once in the early summer and again in the fall You are called one hot summer day to see a group of horses in New Mexico which are slobbering and not eating their hay. Three out of 20 horses seem to be visibly affected. On physical exam of the first one, you find fever of 104 F (40 C) and obvious oral ulcers as shown in the image, mainly on the tongue. What is your tentative diagnosis? A. Bovine papular stomatitis B. Foot-and-Mouth disease (FMD) C. Glanders D. Vesicular stomatitis E. African Horse Sickness - CORRECT ANSWER: D. Vesicular stomatitis You are called one hot summer day to see a group of horses in New Mexico which are slobbering and not eating their hay. Three out of 20 horses seem to be visibly affected. On physical exam of the first one, you find fever of 104 F (40 C) and obvious oral ulcers, mainly on the tongue. What is your tentative diagnosis? A. Glanders B. Vesicular stomatitis C. Foot-and-Mouth disease (FMD) D. Bovine papular stomatitis E. African Horse Sickness - CORRECT ANSWER: B. Vesicular stomatitis You are called out to examine a 4-year old dromedary who was castrated 10-days ago and is now not eating. The groin and prepucial area is extremely swollen. The camel is unable to stand and is drooling. His neck is extended and he appears to be unable to swallow. Which of the following is most likely responsible for these symptoms? A. Rabies virus B. Trypanosoma evansi C. Clostridium tetani D. Methicillin-resistant Staphylococcus aureus - CORRECT ANSWER: C. Clostridium tetani You are called to examine and treat a valuable 3 year old show cow that appeared normal yesterday but has collapsed and is too weak to get up (see photo). You examine her and find T=102F or 38.9 C, HR=130, and RR=42. Her heart is pounding very loudly. The mucous membranes of her eyes, mouth and vulva are all very pale. You diagnose an acutely bleeding abomasal ulcer. What is the most important treatment? A. 20 liters of 1.3% sodium bicarbonate IV B. 40 liters balanced electrolytes orally C. 4 to 8 liters of fresh whole blood IV D. 4 liters of commercial plasma IV plus vitamin K E. 40 liters of saline IV - CORRECT ANSWER: C. 4 to 8 liters of fresh whole blood IV You are called to examine and treat a valuable 3 year old show cow that appeared normal yesterday but has collapsed and is too weak to get up. You examine her and find T=102F or 38.9 C, HR=130, and RR=42. Her heart is pounding very loudly. The mucous membranes of her eyes, mouth and vulva are all very pale. You diagnose an acutely bleeding abomasal ulcer. What is the most important treatment? A. 20 liters of 1.3% sodium bicarbonate IV B. 4 liters of commercial plasma IV plus vitamin K C. 40 liters of saline IV D. 40 liters balanced electrolytes orally E. 4 to 8 liters of fresh whole blood IV - CORRECT ANSWER: E. 4 to 8 liters of fresh whole blood IV You are doing a summer externship in South America and performing physical exams on a variety of animals. You are performing a fundic exam on the eye of a horse and note what appears to be a worm migrating through the conjunctiva. Because of the location and appearance of this parasite, you suspect this is which of the following? A. Oxyuris B. Thelazia C. Toxocara D. Dirofilaria - CORRECT ANSWER: B. Thelazia You are examining a 13-year old Standardbred brood mare 5 hours after parturition. You note that the placenta is still present in the reproductive tract of the mare. What is the most appropriate therapy? A. Administer oxytocin (IM or IV) and lavage the uterus to facilitate removal B. Do nothing; the placenta is not considered retained until 12 hours post-parturition and it will likely be expelled by this time C. Administer penicillin (IM) until the placenta is expelled on its own D. Place physical traction on the placenta and remove it manually - CORRECT ANSWER: A. Administer oxytocin (IM or IV) and lavage the uterus to facilitate removal You are helping a local goat dairy owner interpret the results of semi-annual ELISA serological testing of her herd for caprine arthritis encephalitis (CAE). One 2-month-old kid is CAE-positive by ELISA. Another 2-month-old kid is CAE-negative by ELISA. How do you interpret these results? A. Cannot tell if either kid is truly CAE positive or negative B. CAE-positive kid has CAE, CAE-negative kid does not have CAE C. Both kids are infected with CAE D. CAE-positive kid has CAE, cannot say if CAE-negative kid is infected or not E. Cannot say if CAE-positive kid has CAE, CAE-negative kid is not infected - CORRECT ANSWER: A. Cannot tell if either kid is truly CAE positive or negative -Positive test in kid younger than 90 days old can reflect maternal antibody or false positive; negative test in kid younger than 90 days old does not rule out CAE. You are presented a healthy 10-year-old maiden mare for breeding evaluation in January in North America. On rectal palpation, both ovaries have multiple, small (less than 15 mm) follicles and no corpus lutea. The uterus is flaccid with no edema. Which one of the following choices is the most likely explanation of the findings in this mare? A. Seasonal anestrus B. The mare is in diestrus C. Estrus has just passed D. Early pregnancy E. The mare is too old and now is infertile - CORRECT ANSWER: A. Seasonal anestrus You are presented with a backyard chicken one Saturday at your clinic. The owners report that the hen is laying less eggs than normal and seems listless. You examine the chicken and observe tiny blackish brown specks moving around the vent. Which one of the following is the most likely diagnosis? A. Knemidocoptes mutans B. Argas persicus C. Northern Fowl Mite D. Chicken Mite E. Sticktight flea - CORRECT ANSWER: C. Northern Fowl Mite You are presented with a bitch that whelped her first litter 2 weeks ago. Two of the pups have died and the remaining 2 have neurological signs, suckle poorly, and cry constantly. Which one of the following choices is the primary differential diagnosis? A. Canine herpes virus B. Canine parvovirus C. Neonatal septicemia D. Transplacental roundworm infection E. Congenital disease - CORRECT ANSWER: A. Canine herpes virus You are presented with a cat that is coughing at home, and you perform a bronchoalveolar lavage (BAL) or endotracheal wash. Which cell type, when present in BAL or endotracheal wash cytology, is suggestive of feline asthma? A. Eosinophils B. Basophils C. Neutrophils D. Lymphocytes E. Monocytes - CORRECT ANSWER: A. Eosinophils You are visiting a small "back-yard" flock of chickens and examining the animals. You find several small gray-red mites (approx 0.7 mm) on several of the chickens which you recognize as Dermanyssus gallinae and Ornithonyssus sylviarum. How are these mites best controlled? A. Control of both Dermanyssus gallinae and Ornithonyssus sylviarum relies on thorough insecticidal treatment of the environment

Documents

Summaries

Exam

Lecture notes

Thesis

Study notes

Schemes

Document Store

View all

questions

Latest questions

Biology and Chemistry

Psychology and Sociology

Management

Physics

University

United States of America (USA)

United Kingdom

NAVLE Prep Questions and Verified 500 Correct Answers Guaranteed A+ At the First Attempt | Exams Nursing | Docsity (42)

Sell documents

Seller's Handbook

About us

Career

Contact us

Partners

How does Docsity work

Koofers

Español

Italiano

English

Srpski

Polski

Русский

Português

Français

Deutsch

United Kingdom

United States of America

India

Terms of Use

Cookie Policy

Cookie setup

Privacy Policy

Sitemap Resources

Sitemap Latest Documents

Sitemap Languages and Countries

Copyright © 2024 Ladybird Srl - Via Leonardo da Vinci 16, 10126, Torino, Italy - VAT 10816460017 - All rights reserved

NAVLE Prep Questions and Verified 500 Correct Answers Guaranteed A+ At the First Attempt | Exams Nursing | Docsity (2024)
Top Articles
Latest Posts
Article information

Author: Velia Krajcik

Last Updated:

Views: 5772

Rating: 4.3 / 5 (74 voted)

Reviews: 89% of readers found this page helpful

Author information

Name: Velia Krajcik

Birthday: 1996-07-27

Address: 520 Balistreri Mount, South Armand, OR 60528

Phone: +466880739437

Job: Future Retail Associate

Hobby: Polo, Scouting, Worldbuilding, Cosplaying, Photography, Rowing, Nordic skating

Introduction: My name is Velia Krajcik, I am a handsome, clean, lucky, gleaming, magnificent, proud, glorious person who loves writing and wants to share my knowledge and understanding with you.